Onko evoluutioteoriaa vastaan oikeasti olemassa tieteellisiä

miten on

todisteita?

Raamatun Sanan lisäksi, tottakai D;

Luen ussan yo-kertauskirjaa, ja täällä sanotaan mm.:

"Ensimmäiset biokemialliset ajoitusmenetelmät 1995, makroevoluution matemaattinen mahdottomuus osoitettiin"

"Mikrobiologiset tutkimukset todistavat evoluutioteoriaa vastaan 1990-luvulla"

Yritin etsiä näihin liittyvää tietoa netistä, mut en oikein löytänyt mitään järkevää. Osaako joku luonnontieteisiin perehtynyt kertoa?

107

166

    Vastaukset

    Anonyymi (Kirjaudu / Rekisteröidy)
    5000
    • Uskomatonta. :D

    • Minkä niminen, kenen kirjoittama ja kenen julkaisema tuo kirjasi on?

      • ap,

        Uskonto: lukion kertauskirja, Soili Tiimonen, WSOY

        Ei kyllä ole mikään kaikkein laadukkain teos, sisältää aika paljon kirjoitusvirheitä ja joissain muissakin kohdissa asiat on ilmaistu vähän oudosti...

        Kirjan tekijä on näemmä kirjoittanut Hesariin tällaista settiä: http://www2.hs.fi/uutiset/juttu.asp?id=20020721PA4

        Olisikohan halunnut sitten tuoda omaa vakaumustaan paremmin esille koulukirjoissa :O


      • ap, kirjoitti:

        Uskonto: lukion kertauskirja, Soili Tiimonen, WSOY

        Ei kyllä ole mikään kaikkein laadukkain teos, sisältää aika paljon kirjoitusvirheitä ja joissain muissakin kohdissa asiat on ilmaistu vähän oudosti...

        Kirjan tekijä on näemmä kirjoittanut Hesariin tällaista settiä: http://www2.hs.fi/uutiset/juttu.asp?id=20020721PA4

        Olisikohan halunnut sitten tuoda omaa vakaumustaan paremmin esille koulukirjoissa :O

        Niin. Onhan se surullista luettavaa kun aikuinen ihminen kuten tuo Soili Tiimonen ei erota fantasiaa todellisuudesta.

        Asiaa on tutkittu:

        "Study: Religious children are less able to distinguish fantasy from reality

        ...Although this might be unsurprising, secular and religious children also differed in their interpretation of fantasy narratives where there was a supernatural or magical storyline.

        "Secular children were more likely than religious children to judge the protagonist in such fantastical stories to be fictional," wrote the researchers.

        "The results suggest that exposure to religious ideas has a powerful impact on children's differentiation between reality and fiction, not just for religious stories but also for fantastical stories." "

        http://www.bbc.com/news/blogs-echochambers-28537149

        Ei siis tarvi ihmetellä miksi Tiimonen pitää esim. Harry Potteria ja pääsiäisnoitia uhkana nykynuorille.


      • ap, kirjoitti:

        Uskonto: lukion kertauskirja, Soili Tiimonen, WSOY

        Ei kyllä ole mikään kaikkein laadukkain teos, sisältää aika paljon kirjoitusvirheitä ja joissain muissakin kohdissa asiat on ilmaistu vähän oudosti...

        Kirjan tekijä on näemmä kirjoittanut Hesariin tällaista settiä: http://www2.hs.fi/uutiset/juttu.asp?id=20020721PA4

        Olisikohan halunnut sitten tuoda omaa vakaumustaan paremmin esille koulukirjoissa :O

        Aika hupaisaa kun hihhuli sanoo "Kriittinen kuluttaja tunnistaa rahastuksen maun"...


      • ap, kirjoitti:

        Uskonto: lukion kertauskirja, Soili Tiimonen, WSOY

        Ei kyllä ole mikään kaikkein laadukkain teos, sisältää aika paljon kirjoitusvirheitä ja joissain muissakin kohdissa asiat on ilmaistu vähän oudosti...

        Kirjan tekijä on näemmä kirjoittanut Hesariin tällaista settiä: http://www2.hs.fi/uutiset/juttu.asp?id=20020721PA4

        Olisikohan halunnut sitten tuoda omaa vakaumustaan paremmin esille koulukirjoissa :O

        Huomattakoon kuitenkin, että Soili kehuu Skepsiksen toimintaa! Oletteko koskaan kuulleet palstakretujen sanovan siitä edes myönteisen sanan puolikasta?


    • Immured_soul

      Ei varsinaisesti ehkä vastaan, mutta evoluutioteoriassa on toki aukkoja. Kuten nyt kaiken elämän alku, johon on useita teorioita ja jota ei olla pystytty todistamaan. Myös se ihmisapinan ja ihmisen välinen linkki puuttuu, eli sellaista apinaihmisoliota ei olla löydetty, josta selkeästi olisi myöhemmin kehittynyt ihminen.

      Tämä jättää tilaa kaikenlaisille uskomuksille Jumalasta avaruusolioiden geeenimanipulaatioon...oikeastaan hyvä niin.

      • *Ei varsinaisesti ehkä vastaan, mutta evoluutioteoriassa on toki aukkoja. Kuten nyt kaiken elämän alku, johon on useita teorioita ja jota ei olla pystytty todistamaan.*

        Evoluutioteorialla viitataan biologiseen evoluutioon, joka ei yritäkään selittää elämän syntyä. Evoluutioteoriat kuten synteettinen evoluutioteria selittävät miten luonnossa havaittava ilmiö nimeltä evoluutio toimii.

        *Myös se ihmisapinan ja ihmisen välinen linkki puuttuu, eli sellaista apinaihmisoliota ei olla löydetty, josta selkeästi olisi myöhemmin kehittynyt ihminen.*

        Ei ole löytynyt vain niiden mielestä jotka haluavat asian kieltää tai eivät asiaa muuten ymmärrä. Miljoonia vuosia sitten eläneistä eliöistä ei ole jäljellä kuin enintään luita eikä usein niitäkään vaan niiden kivettyneitä jäljenteitä, fossiileja. Todisteita kehityslinjoista on silti yllin kyllin, ja lisää löytyy vuosi vuodelta.

        http://fi.wikipedia.org/wiki/Ihmisen_evoluutio#mediaviewer/Tiedosto:Ihmisten_sukupuu3_vuosi2004.jpg

        Sitä mukaa kun fossiileja löytyy lisää, edelläviitattu sukupuu tarkentuu. Eivätkä fossiilit suinkaan ole ainoat todisteet lajien sukulaisuuden puolesta: nykylajeja tutkimalla voidaan tehdä havaintoja joiden perusteella voidaan tehdä päätelmiä menneestä ajasta.


      • Råger Mer
        bg-ope kirjoitti:

        *Ei varsinaisesti ehkä vastaan, mutta evoluutioteoriassa on toki aukkoja. Kuten nyt kaiken elämän alku, johon on useita teorioita ja jota ei olla pystytty todistamaan.*

        Evoluutioteorialla viitataan biologiseen evoluutioon, joka ei yritäkään selittää elämän syntyä. Evoluutioteoriat kuten synteettinen evoluutioteria selittävät miten luonnossa havaittava ilmiö nimeltä evoluutio toimii.

        *Myös se ihmisapinan ja ihmisen välinen linkki puuttuu, eli sellaista apinaihmisoliota ei olla löydetty, josta selkeästi olisi myöhemmin kehittynyt ihminen.*

        Ei ole löytynyt vain niiden mielestä jotka haluavat asian kieltää tai eivät asiaa muuten ymmärrä. Miljoonia vuosia sitten eläneistä eliöistä ei ole jäljellä kuin enintään luita eikä usein niitäkään vaan niiden kivettyneitä jäljenteitä, fossiileja. Todisteita kehityslinjoista on silti yllin kyllin, ja lisää löytyy vuosi vuodelta.

        http://fi.wikipedia.org/wiki/Ihmisen_evoluutio#mediaviewer/Tiedosto:Ihmisten_sukupuu3_vuosi2004.jpg

        Sitä mukaa kun fossiileja löytyy lisää, edelläviitattu sukupuu tarkentuu. Eivätkä fossiilit suinkaan ole ainoat todisteet lajien sukulaisuuden puolesta: nykylajeja tutkimalla voidaan tehdä havaintoja joiden perusteella voidaan tehdä päätelmiä menneestä ajasta.

        On se vaan hienoa huomata että vielä aikuisetkin ihmiset uskovat satuihin jota evoluutioteoria ja Tuhkimo-satu saumattomasti edustaa.


      • Råger Mer kirjoitti:

        On se vaan hienoa huomata että vielä aikuisetkin ihmiset uskovat satuihin jota evoluutioteoria ja Tuhkimo-satu saumattomasti edustaa.

        Tuhkimo-satu on hyvä ja osoittaa miltä osin luonnonvalinta, geneettinen ajautuminen, mutaatiot ja rekombinaatio ovat yliluonnollisia asioita / mielikuvitusta / satua.

        Tuhkimo-satu kykenee kyllä trollaamaan tuota idiotismiaan pitkin nettiä, muttei tuottamaan jälkeläistä joka olisi klooni hänestä itsestään.

        Ja koska ei voi - toteuttaa tietämättään evoluutiota.

        Maailma muuttuu, trolliseni.


      • Råger Mer
        bg-ope kirjoitti:

        Tuhkimo-satu on hyvä ja osoittaa miltä osin luonnonvalinta, geneettinen ajautuminen, mutaatiot ja rekombinaatio ovat yliluonnollisia asioita / mielikuvitusta / satua.

        Tuhkimo-satu kykenee kyllä trollaamaan tuota idiotismiaan pitkin nettiä, muttei tuottamaan jälkeläistä joka olisi klooni hänestä itsestään.

        Ja koska ei voi - toteuttaa tietämättään evoluutiota.

        Maailma muuttuu, trolliseni.

        Sitähän ne Tuhkimo-satu ja evoluutio ovat eli satuja. Mutta onhan ne jänskempiä kun niihin uskoo.


      • ei luottoa sulle
        bg-ope kirjoitti:

        *Ei varsinaisesti ehkä vastaan, mutta evoluutioteoriassa on toki aukkoja. Kuten nyt kaiken elämän alku, johon on useita teorioita ja jota ei olla pystytty todistamaan.*

        Evoluutioteorialla viitataan biologiseen evoluutioon, joka ei yritäkään selittää elämän syntyä. Evoluutioteoriat kuten synteettinen evoluutioteria selittävät miten luonnossa havaittava ilmiö nimeltä evoluutio toimii.

        *Myös se ihmisapinan ja ihmisen välinen linkki puuttuu, eli sellaista apinaihmisoliota ei olla löydetty, josta selkeästi olisi myöhemmin kehittynyt ihminen.*

        Ei ole löytynyt vain niiden mielestä jotka haluavat asian kieltää tai eivät asiaa muuten ymmärrä. Miljoonia vuosia sitten eläneistä eliöistä ei ole jäljellä kuin enintään luita eikä usein niitäkään vaan niiden kivettyneitä jäljenteitä, fossiileja. Todisteita kehityslinjoista on silti yllin kyllin, ja lisää löytyy vuosi vuodelta.

        http://fi.wikipedia.org/wiki/Ihmisen_evoluutio#mediaviewer/Tiedosto:Ihmisten_sukupuu3_vuosi2004.jpg

        Sitä mukaa kun fossiileja löytyy lisää, edelläviitattu sukupuu tarkentuu. Eivätkä fossiilit suinkaan ole ainoat todisteet lajien sukulaisuuden puolesta: nykylajeja tutkimalla voidaan tehdä havaintoja joiden perusteella voidaan tehdä päätelmiä menneestä ajasta.

        Valitettavasti päätelmiä ei voi pitää todisteina!
        Yritä uudelleen.


      • +++++++++++++

        "Kuten nyt kaiken elämän alku, johon on useita teorioita ja jota ei olla pystytty todistamaan. "

        Ei liity evoluutioteoriaan.

        "Myös se ihmisapinan ja ihmisen välinen linkki puuttuu, eli sellaista apinaihmisoliota ei olla löydetty, josta selkeästi olisi myöhemmin kehittynyt ihminen."

        Ihminen ON Ihmisapina.


      • Råger Mer kirjoitti:

        Sitähän ne Tuhkimo-satu ja evoluutio ovat eli satuja. Mutta onhan ne jänskempiä kun niihin uskoo.

        *Sitähän ne Tuhkimo-satu ja evoluutio ovat eli satuja. Mutta onhan ne jänskempiä kun niihin uskoo.*

        Sinusta ei siis - yllätys - ollut kertomaan miltä osin evoluutio on satua.

        Mitäpä jos nyt toistaisit tuota tyhjää mantraasi vielä sen parikymmentä kertaa lisää, niin todistat todella olevasi yksinkertainen, keskusteluun kykenemätön trolli?


      • jep jep

        "Ei varsinaisesti ehkä vastaan, mutta evoluutioteoriassa on toki aukkoja. Kuten nyt kaiken elämän alku"

        koettakaa jo ymmärtää, evoluutio, ja sitä selittävä evoluutioteoria ei käsittele millään tavalla elämän alkua, sille se on yks ja sama mistä elämä on tullut.

        Ainoa asia jota evoluutio ja evoluutioteoria sisältää, on jo syntyneen elämän muotojen muutoksia.

        Kuten charles darwinin kirjan otsikkoki sen sanoi "LAJIEN synty"


      • bg-ope kirjoitti:

        *Sitähän ne Tuhkimo-satu ja evoluutio ovat eli satuja. Mutta onhan ne jänskempiä kun niihin uskoo.*

        Sinusta ei siis - yllätys - ollut kertomaan miltä osin evoluutio on satua.

        Mitäpä jos nyt toistaisit tuota tyhjää mantraasi vielä sen parikymmentä kertaa lisää, niin todistat todella olevasi yksinkertainen, keskusteluun kykenemätön trolli?

        >Mitäpä jos nyt toistaisit tuota tyhjää mantraasi vielä sen parikymmentä kertaa lisää, niin todistat todella olevasi yksinkertainen, keskusteluun kykenemätön trolli?

        Eiköhän tuo ole jo todistettu tieteellisen pätevästi.

        Kummasti täällä palstalla alkaa ajoittain miettiä, olisiko se sittenkään niin paha juttu jos joitakin toimettomia työttömiä alettaisiin tuuppia suomi24.n ääreltä pakkotyöhön.


      • ei luottoa sulle kirjoitti:

        Valitettavasti päätelmiä ei voi pitää todisteina!
        Yritä uudelleen.

        Tieteelliset päätelmät perustuvat todisteisiin ja ovat analyyttisia yhteenvetoja niistä.

        Älä suotta yritä uudelleen.


      • >Kuten nyt kaiken elämän alku, johon on useita teorioita ja jota ei olla pystytty todistamaan.

        Hypoteeseja, ei teorioita. Teoria tulee sitten kun alkaa olla kovaa näyttöä.


      • +d7g9sdfobg
        jep jep kirjoitti:

        "Ei varsinaisesti ehkä vastaan, mutta evoluutioteoriassa on toki aukkoja. Kuten nyt kaiken elämän alku"

        koettakaa jo ymmärtää, evoluutio, ja sitä selittävä evoluutioteoria ei käsittele millään tavalla elämän alkua, sille se on yks ja sama mistä elämä on tullut.

        Ainoa asia jota evoluutio ja evoluutioteoria sisältää, on jo syntyneen elämän muotojen muutoksia.

        Kuten charles darwinin kirjan otsikkoki sen sanoi "LAJIEN synty"

        Aivan, kirjan nimi ei ole "ENSIMMÄISEN LAJIN SYNTY".


      • Huijausta
        bg-ope kirjoitti:

        *Ei varsinaisesti ehkä vastaan, mutta evoluutioteoriassa on toki aukkoja. Kuten nyt kaiken elämän alku, johon on useita teorioita ja jota ei olla pystytty todistamaan.*

        Evoluutioteorialla viitataan biologiseen evoluutioon, joka ei yritäkään selittää elämän syntyä. Evoluutioteoriat kuten synteettinen evoluutioteria selittävät miten luonnossa havaittava ilmiö nimeltä evoluutio toimii.

        *Myös se ihmisapinan ja ihmisen välinen linkki puuttuu, eli sellaista apinaihmisoliota ei olla löydetty, josta selkeästi olisi myöhemmin kehittynyt ihminen.*

        Ei ole löytynyt vain niiden mielestä jotka haluavat asian kieltää tai eivät asiaa muuten ymmärrä. Miljoonia vuosia sitten eläneistä eliöistä ei ole jäljellä kuin enintään luita eikä usein niitäkään vaan niiden kivettyneitä jäljenteitä, fossiileja. Todisteita kehityslinjoista on silti yllin kyllin, ja lisää löytyy vuosi vuodelta.

        http://fi.wikipedia.org/wiki/Ihmisen_evoluutio#mediaviewer/Tiedosto:Ihmisten_sukupuu3_vuosi2004.jpg

        Sitä mukaa kun fossiileja löytyy lisää, edelläviitattu sukupuu tarkentuu. Eivätkä fossiilit suinkaan ole ainoat todisteet lajien sukulaisuuden puolesta: nykylajeja tutkimalla voidaan tehdä havaintoja joiden perusteella voidaan tehdä päätelmiä menneestä ajasta.

        Synteettinen evoluutioteoria esittää uskomuksen, tai oikeammin toivomuksen siitä että miten evoluution haluttaisiin toimivan. Sääli vain että ei tiede, eikä edes uskomuksistaan tunnettu epä-Skepsis ei löydä mitään todisteita evoluution puolesta.


      • Evoankka
        jep jep kirjoitti:

        "Ei varsinaisesti ehkä vastaan, mutta evoluutioteoriassa on toki aukkoja. Kuten nyt kaiken elämän alku"

        koettakaa jo ymmärtää, evoluutio, ja sitä selittävä evoluutioteoria ei käsittele millään tavalla elämän alkua, sille se on yks ja sama mistä elämä on tullut.

        Ainoa asia jota evoluutio ja evoluutioteoria sisältää, on jo syntyneen elämän muotojen muutoksia.

        Kuten charles darwinin kirjan otsikkoki sen sanoi "LAJIEN synty"

        Aivan, se ensimmäinen alkuliemessä muhinut alkusolu ei ollutkaan mikään laji, mutta oliko se siitä kaksisoluiseksi mutatoitunut jo laji? Olipa tietenkin, mutta miksi evoluutioteoria ei tiedä siitäkään yhtään mitään? Eikä se edes sitä osaa selittää että mikä se varsinaisesti ensimmäinen laji oli joka lasketaan evoluution aluksi. Eikä se osaa selittää sitäkään että missä ja miten ja milloin erityivät tuosta alusta erilaiset kasvit, hyönteiset, sienet, eläimet jne.

        Siis mitä se evoluutioteoria oikeastaan tietää? Eikö yhtään mitään? Eipä tietenkään.


      • Huijausta kirjoitti:

        Synteettinen evoluutioteoria esittää uskomuksen, tai oikeammin toivomuksen siitä että miten evoluution haluttaisiin toimivan. Sääli vain että ei tiede, eikä edes uskomuksistaan tunnettu epä-Skepsis ei löydä mitään todisteita evoluution puolesta.

        Älä tyttöseni suotta sääli, tiedeyhteisö on kyllä hyvin tyytyväinen lukemattomiin toistaiseksi löytyneisiin todisteisiin. Jopa jokseenkin kaikki kristitytkin tiedemiehet ovat oikein tyytyväisiä.

        Suurta tyytymättömyyttä esiintyy kyllä erilaisissa uskonlahkoissa, kun kouluja käymättömät lahkolaiset pelkäävät kuollakseen joskus ymmärtävänsä että he ovatkin vain apinoita.


      • Evoankka kirjoitti:

        Aivan, se ensimmäinen alkuliemessä muhinut alkusolu ei ollutkaan mikään laji, mutta oliko se siitä kaksisoluiseksi mutatoitunut jo laji? Olipa tietenkin, mutta miksi evoluutioteoria ei tiedä siitäkään yhtään mitään? Eikä se edes sitä osaa selittää että mikä se varsinaisesti ensimmäinen laji oli joka lasketaan evoluution aluksi. Eikä se osaa selittää sitäkään että missä ja miten ja milloin erityivät tuosta alusta erilaiset kasvit, hyönteiset, sienet, eläimet jne.

        Siis mitä se evoluutioteoria oikeastaan tietää? Eikö yhtään mitään? Eipä tietenkään.

        *Eikä se osaa selittää sitäkään että missä ja miten ja milloin erityivät tuosta alusta erilaiset kasvit, hyönteiset, sienet, eläimet jne. *

        Osaa. Tietämättömyytesi asioista on todiste tietämättömyydestäsi, ei muuta.


      • *JC
        RepeRuutikallo kirjoitti:

        Älä tyttöseni suotta sääli, tiedeyhteisö on kyllä hyvin tyytyväinen lukemattomiin toistaiseksi löytyneisiin todisteisiin. Jopa jokseenkin kaikki kristitytkin tiedemiehet ovat oikein tyytyväisiä.

        Suurta tyytymättömyyttä esiintyy kyllä erilaisissa uskonlahkoissa, kun kouluja käymättömät lahkolaiset pelkäävät kuollakseen joskus ymmärtävänsä että he ovatkin vain apinoita.

        "...pelkäävät kuollakseen joskus ymmärtävänsä että he ovatkin vain apinoita."

        Sitä päivää pelkään todellakin kuollakseni. Olisin silloin menettänyt järkeni, jos yhtäkkiä luulisin olevani apina tai edes apinan sukua.

        Yhtäkään ihmistä, edes höperöintä evoa, en voisi loukata sanomalla: olet apina. Jokainen ihminen, uskomuksistaan riippumatta, on Aadamin ja Evan sukua, Jumalan omaksi kuvakseen luoman ihmisen sukua.

        Apina on taas luotu Afrikan viidakoihin ja savanneille, elämään elämäänsä muiden maaeläinten tavoin. Eri apinalajeja on lukuisia, toki muuallakin kuin Afrikassa niitä elää.

        Ihmisiä on vain yksi laji kautta koko maailman, ihminen.


      • *JC kirjoitti:

        "...pelkäävät kuollakseen joskus ymmärtävänsä että he ovatkin vain apinoita."

        Sitä päivää pelkään todellakin kuollakseni. Olisin silloin menettänyt järkeni, jos yhtäkkiä luulisin olevani apina tai edes apinan sukua.

        Yhtäkään ihmistä, edes höperöintä evoa, en voisi loukata sanomalla: olet apina. Jokainen ihminen, uskomuksistaan riippumatta, on Aadamin ja Evan sukua, Jumalan omaksi kuvakseen luoman ihmisen sukua.

        Apina on taas luotu Afrikan viidakoihin ja savanneille, elämään elämäänsä muiden maaeläinten tavoin. Eri apinalajeja on lukuisia, toki muuallakin kuin Afrikassa niitä elää.

        Ihmisiä on vain yksi laji kautta koko maailman, ihminen.

        >Yhtäkään ihmistä, edes höperöintä evoa, en voisi loukata sanomalla: olet apina.

        Loukkauksen onnistuminen edellyttää että kohde kokee sen loukkauksena. Koska tiedän edustavani lajia joka on apinoista kehittynein ja on itse asian selvittänyt pätevimmillä tutkimuksillaan, en ymmärrä miksi loukkaantuisin.

        Ja samaa lajia muuten edustat sinäkin.:)


    • Iof-id

      Evoluutiouskonto, ateismiuskonto ja islamuskonto perustuvat Tuhkimosatuun. Tämän vahvistavat Toni T, Jaakob ja JC.

      • Kas, esitin saman järkevän kysymyksen kuin yksi Demin kirjoittaja.

        Luettuani tuon Demin ketjun alan jotenkin epäillä että vastausta ei tipu...

        Mutta eipä se mitään, aloittaja on osoittautunut trolliksi ja trolleja on lupa lyödä kuin vierasta sikaa...


    • ninjax.
      • Heh !

        >

        Minäpä luin. Oikeastaan oli aika humoristista tekstiä, jos miettii sitä kokonaisuutena.

        Iivanainen väittää tieteellistä katsomusta uskonnoksi. Ihan yhtä paljon uskonnoksi kuin vaikkapa kristinuskoa. Kuitenkin hän sitten toisaalla perustelee Raamatun oikeellisuutta sillä, että Raamatussa mainituista paikoista ja henkilöistä on ihan aitoja todisteita. Ne ovat (osin) todisteellisia asioita, joista todisteista voidaan vetää vieläpä looginen johtopäätelmä, että jotkut Raamatussa mainitut tapahtumat on ihan aidosti tapahtuneet (hän kyllä auliisti myöntää, että suurin osa Raamatun tapahtumista on täysin todisteettomia uskomuksia). Tässäpä vaan Iivanainen käyttää nimenomaan tieteellistä todistustapaa, jota hän pitää tapahtuman oikeaksi todistavana - eikä siis tiedettä ollenkaan enää uskontona (yhtä vailla pohjaa kuin kristinuskolla) vaan nimenomaan uskomuksen poistajana. Sitten kuitenkin hän palaa taas samaan väitteeseensä tieteen vertautumisesta uskontoon kun kyseessä on vaikkapa ihan samanlaiset luonnosta löytyvät todisteet alkuräjähdyksestä kuin vaikka Raamatussa mainitun kaupungin muurit raunioista erämaassa. Toisessa kohdassa siis luonnosta löytyvä todiste osoittaa tapahtuneen todeksi mutta mikäli se on Iivanaisen uskontoa vastaan, samanlainen looginen todisterypäs onkin hänen mielestään uskontoa.

        Aika vekkuli kaksoisstandardi vai mitä ?

        Ja tosiaan, harvapa pitää painovoimateoriaa uskontona (vaikka Iivanainen miten selittäisi, että tiede on uskontoa). Onko se Sinusta uskonto ? Onko tiede uskontoa ?

        Sitten toinen usein toistuva kreationistien oman jalan sahaaminen: "abiogeneesistä ei ole todisteita, joten pitää vaan uskoa, että elämä on syntynyt ilman jumalallista väliintuloa". No, näinhän se toisaalta onkin, että koska abiogeneesista on vain osin todistettuja osaprosesseja, kyse on vain hypoteesista emmekä voi pitää sitä todistettuna asiana. [Ei meillä tosin ole mitään perustetta olettaa jotain yliluonnollistakaan abiogeneesiin.] Mutta mutta, tässä piilee se kreationismin murskaava seikka, että vaikka abiogeneesi olisikin tuntematon, todisteeton (ja jopa mahdollisesti yliluonnollinen), niin evoluutio on sitten taas todisteellinen ja luonnollinen prosessi. Tässä siis Iivanainen myöntää loogisella askeleella, että esim. ihminen on simpanssin lähisukulainen (ja kaikki eliöt on polveutuneet LUCA:sta), koska se on sitten taas jo todisteellinen juttu.

        Niin, miten suu nyt pannaan ?


    • ymmärtäkää.

      Niin se kuitenkin on, että matematiikka on kaikkien luonnontieteiden isoäiti, joten evoluutioteoria saa väistyä kun kerran on matemaattinen todistus sitä vastaan.

      • Heh !

        >

        Köh tuota, eihän matematiikka ole edes luonnontiede.

        >

        Olet tietysti oikeassa siinä, että jos matemaattisesti osoitetaan, että evoluutioteoria/painovoimateoria/suhteellisuusteoria tms. on mahdoton, ko. teoria saa väistyä. Se murskautuu välttämättäkin matematiikan edessä. Nyt vaan pitäisi löytää se aukoton matemaattinen todistus, että evoluutioteoria olisi mahdoton. Eipä sellaista luonnollisesti ole olemassa, joten evoluutioteoria on kiistaton edelleen.

        Voi voi.


      • makro = hoax
        Heh ! kirjoitti:

        >

        Köh tuota, eihän matematiikka ole edes luonnontiede.

        >

        Olet tietysti oikeassa siinä, että jos matemaattisesti osoitetaan, että evoluutioteoria/painovoimateoria/suhteellisuusteoria tms. on mahdoton, ko. teoria saa väistyä. Se murskautuu välttämättäkin matematiikan edessä. Nyt vaan pitäisi löytää se aukoton matemaattinen todistus, että evoluutioteoria olisi mahdoton. Eipä sellaista luonnollisesti ole olemassa, joten evoluutioteoria on kiistaton edelleen.

        Voi voi.

        >

        Matematiikka on kaikkien luonnontieteiden perusta.

        >

        Voi voi, kun tässä on nyt vastakkain parhaaseen tietoon perustuva lukiokirja ja anonyymi suomi24 kirjoittaja. Todistappa että sellaista osoitusta ei siis ole.


      • makro = hoax kirjoitti:

        >

        Matematiikka on kaikkien luonnontieteiden perusta.

        >

        Voi voi, kun tässä on nyt vastakkain parhaaseen tietoon perustuva lukiokirja ja anonyymi suomi24 kirjoittaja. Todistappa että sellaista osoitusta ei siis ole.

        *Voi voi, kun tässä on nyt vastakkain parhaaseen tietoon perustuva lukiokirja*

        Uskovaisen uskonsa dogmien pohjalta koostama uskonto-aiheinen kirja ei TODELLAKAAN ole parasta tietoa mistään, VARSINKAAN biologiasta.

        *Todistappa että sellaista osoitusta ei siis ole.*

        Evoluution (eli luonnossa ilmenevän faktuaalisen ilmiön) osoittaminen olemattomaksi on juuri yhtä järkevää kuin osoittaa ettei vesisadetta ole olemassa. Tiede pystyy selittämään MITEN vesisade syntyy ja MITKÄ tekijät sen aiheuttavat. Ilmiölle on siis olemassa selitys eli teoria. Sama koskee evoluutiota.

        Mikä olikaan Raamatun selitys vesisateelle? Ai niin, jotain että GDI ja muuta ei ole tarpeen asiasta tietää. Kun eivät kamelikuskitkaan tienneet.


      • epäonnistuit
        bg-ope kirjoitti:

        *Voi voi, kun tässä on nyt vastakkain parhaaseen tietoon perustuva lukiokirja*

        Uskovaisen uskonsa dogmien pohjalta koostama uskonto-aiheinen kirja ei TODELLAKAAN ole parasta tietoa mistään, VARSINKAAN biologiasta.

        *Todistappa että sellaista osoitusta ei siis ole.*

        Evoluution (eli luonnossa ilmenevän faktuaalisen ilmiön) osoittaminen olemattomaksi on juuri yhtä järkevää kuin osoittaa ettei vesisadetta ole olemassa. Tiede pystyy selittämään MITEN vesisade syntyy ja MITKÄ tekijät sen aiheuttavat. Ilmiölle on siis olemassa selitys eli teoria. Sama koskee evoluutiota.

        Mikä olikaan Raamatun selitys vesisateelle? Ai niin, jotain että GDI ja muuta ei ole tarpeen asiasta tietää. Kun eivät kamelikuskitkaan tienneet.

        Toisen uskoon viittaaminen ei käy todisteeksi.
        Oletko menettänyt järkesi?


      • eiksjeh?
        bg-ope kirjoitti:

        *Voi voi, kun tässä on nyt vastakkain parhaaseen tietoon perustuva lukiokirja*

        Uskovaisen uskonsa dogmien pohjalta koostama uskonto-aiheinen kirja ei TODELLAKAAN ole parasta tietoa mistään, VARSINKAAN biologiasta.

        *Todistappa että sellaista osoitusta ei siis ole.*

        Evoluution (eli luonnossa ilmenevän faktuaalisen ilmiön) osoittaminen olemattomaksi on juuri yhtä järkevää kuin osoittaa ettei vesisadetta ole olemassa. Tiede pystyy selittämään MITEN vesisade syntyy ja MITKÄ tekijät sen aiheuttavat. Ilmiölle on siis olemassa selitys eli teoria. Sama koskee evoluutiota.

        Mikä olikaan Raamatun selitys vesisateelle? Ai niin, jotain että GDI ja muuta ei ole tarpeen asiasta tietää. Kun eivät kamelikuskitkaan tienneet.

        Miksi teet joka päivä järjenvastaisia asioita? Mietippä sitä.
        Oletko menettänyt järkesi vai eikö sitä koskaan ollutkaan?
        Onko sinulla ollut elämässäsi yhtäkään päivää, että kaikki mitä olet tehnyt ja sanonut, olisivat lleet järkeviä? Minä pelkään pahinta!


      • epäonnistuit kirjoitti:

        Toisen uskoon viittaaminen ei käy todisteeksi.
        Oletko menettänyt järkesi?

        *Toisen uskoon viittaaminen ei käy todisteeksi.
        Oletko menettänyt järkesi?*

        Minusta on oikein järkevää kyseenalaistaa uskonnon kirja biologiaa koskevan tiedon lähteenä, varsinkin kun kirjalla on subjektiivinen motivaatio kyseenalaistaa biologiasta vain ja ainoastaan ne tiedot jotka ovat kirjan kirjoittajan henk.koht. uskonnollisen maailmankuvan vastaisia.

        Olisi mielenkiintoista tietää vastustaako Tiimonen myös esim. teoriaa siitä miten puut saavat nostettua veden juuriltaan latvaan. Liekö tuokin tieto perkeleestä, ja olisi parempi tyytyä vain siihen tuttuun ja turvalliseen "Jeesus nostaa veden puiden juurista latvaan, turha on ihmisen yrittää selvittää asiaa tarkemmin"?


      • eiksjeh? kirjoitti:

        Miksi teet joka päivä järjenvastaisia asioita? Mietippä sitä.
        Oletko menettänyt järkesi vai eikö sitä koskaan ollutkaan?
        Onko sinulla ollut elämässäsi yhtäkään päivää, että kaikki mitä olet tehnyt ja sanonut, olisivat lleet järkeviä? Minä pelkään pahinta!

        *Miksi teet joka päivä järjenvastaisia asioita? Mietippä sitä.
        Oletko menettänyt järkesi vai eikö sitä koskaan ollutkaan?
        Onko sinulla ollut elämässäsi yhtäkään päivää, että kaikki mitä olet tehnyt ja sanonut, olisivat lleet järkeviä? Minä pelkään pahinta!*

        Jos haluat keskustella kanssani, ensimmäinen vaihe on kirjoittaa viesti jossa on jotain tolkkua.


      • epätietone
        bg-ope kirjoitti:

        *Toisen uskoon viittaaminen ei käy todisteeksi.
        Oletko menettänyt järkesi?*

        Minusta on oikein järkevää kyseenalaistaa uskonnon kirja biologiaa koskevan tiedon lähteenä, varsinkin kun kirjalla on subjektiivinen motivaatio kyseenalaistaa biologiasta vain ja ainoastaan ne tiedot jotka ovat kirjan kirjoittajan henk.koht. uskonnollisen maailmankuvan vastaisia.

        Olisi mielenkiintoista tietää vastustaako Tiimonen myös esim. teoriaa siitä miten puut saavat nostettua veden juuriltaan latvaan. Liekö tuokin tieto perkeleestä, ja olisi parempi tyytyä vain siihen tuttuun ja turvalliseen "Jeesus nostaa veden puiden juurista latvaan, turha on ihmisen yrittää selvittää asiaa tarkemmin"?

        No milläs puut nostavat veden latvaan?

        Ja millä puut laskevat nesteen juuriin syksyllä?


      • epätietone kirjoitti:

        No milläs puut nostavat veden latvaan?

        Ja millä puut laskevat nesteen juuriin syksyllä?

        *No milläs puut nostavat veden latvaan?*

        Vesimolekyylit muodostavat yhtenäisiä jonoja juurista lehtiin / neulasiin puun rungon vesiputkien, lehtisuonien ym. kautta. "Vesihissin" laittaa käyntiin aurinko, joka kuumentaa lehden pintaa ja saa veden haihtumaan ilmaan. Tämä saa aikaan imun jota tukee ohuissa vesiputkissa tapahtuva kapillaari-ilmiö: ilmaan haihtuva molekyyli kiskaisee edellistä mukanaan ylöspäin vesiputkessa. Ketjureaktio yltää aina juuren kärkeen saakka, jonne syntyy imu: puu saa imettyä vettä ja sen mukana ravinteita maasta.

        Kapillaari-ilmiö "vesihissi" selittävät myös aamukasteen.

        http://fi.m.wikipedia.org/wiki/Kapillaari-ilmiö

        *Ja millä puut laskevat nesteen juuriin syksyllä?*

        Painovoima hoitaa liikennettä alaspäin.


      • makro = hoax kirjoitti:

        >

        Matematiikka on kaikkien luonnontieteiden perusta.

        >

        Voi voi, kun tässä on nyt vastakkain parhaaseen tietoon perustuva lukiokirja ja anonyymi suomi24 kirjoittaja. Todistappa että sellaista osoitusta ei siis ole.

        >Voi voi, kun tässä on nyt vastakkain parhaaseen tietoon perustuva lukiokirja ja anonyymi suomi24 kirjoittaja.

        Parhaaseen biologian tietoon perustuva ussankirja? Ja lehmät lentää. :D


      • qyfuyfquy
        bg-ope kirjoitti:

        *Miksi teet joka päivä järjenvastaisia asioita? Mietippä sitä.
        Oletko menettänyt järkesi vai eikö sitä koskaan ollutkaan?
        Onko sinulla ollut elämässäsi yhtäkään päivää, että kaikki mitä olet tehnyt ja sanonut, olisivat lleet järkeviä? Minä pelkään pahinta!*

        Jos haluat keskustella kanssani, ensimmäinen vaihe on kirjoittaa viesti jossa on jotain tolkkua.

        Evoluutio-uskovaisilla ei ole mitään tolkkua, on vain sokeaa uskonhurmiota.


      • bg-ope kirjoitti:

        *No milläs puut nostavat veden latvaan?*

        Vesimolekyylit muodostavat yhtenäisiä jonoja juurista lehtiin / neulasiin puun rungon vesiputkien, lehtisuonien ym. kautta. "Vesihissin" laittaa käyntiin aurinko, joka kuumentaa lehden pintaa ja saa veden haihtumaan ilmaan. Tämä saa aikaan imun jota tukee ohuissa vesiputkissa tapahtuva kapillaari-ilmiö: ilmaan haihtuva molekyyli kiskaisee edellistä mukanaan ylöspäin vesiputkessa. Ketjureaktio yltää aina juuren kärkeen saakka, jonne syntyy imu: puu saa imettyä vettä ja sen mukana ravinteita maasta.

        Kapillaari-ilmiö "vesihissi" selittävät myös aamukasteen.

        http://fi.m.wikipedia.org/wiki/Kapillaari-ilmiö

        *Ja millä puut laskevat nesteen juuriin syksyllä?*

        Painovoima hoitaa liikennettä alaspäin.

        Perähikiä on kova mainostamaan luonnonmukaisuuttaan, mutta luonnon toiminnan prosessit ovat näköjään menneet vähän ohi.


      • eiksjeh?
        qyfuyfquy kirjoitti:

        Evoluutio-uskovaisilla ei ole mitään tolkkua, on vain sokeaa uskonhurmiota.

        Miksi kirjoitat joka päivä järjenvastaisia viestejä? Mietippä sitä.


      • bg-ope kirjoitti:

        *No milläs puut nostavat veden latvaan?*

        Vesimolekyylit muodostavat yhtenäisiä jonoja juurista lehtiin / neulasiin puun rungon vesiputkien, lehtisuonien ym. kautta. "Vesihissin" laittaa käyntiin aurinko, joka kuumentaa lehden pintaa ja saa veden haihtumaan ilmaan. Tämä saa aikaan imun jota tukee ohuissa vesiputkissa tapahtuva kapillaari-ilmiö: ilmaan haihtuva molekyyli kiskaisee edellistä mukanaan ylöspäin vesiputkessa. Ketjureaktio yltää aina juuren kärkeen saakka, jonne syntyy imu: puu saa imettyä vettä ja sen mukana ravinteita maasta.

        Kapillaari-ilmiö "vesihissi" selittävät myös aamukasteen.

        http://fi.m.wikipedia.org/wiki/Kapillaari-ilmiö

        *Ja millä puut laskevat nesteen juuriin syksyllä?*

        Painovoima hoitaa liikennettä alaspäin.

        Hississä...

        https://www.youtube.com/watch?v=_3ho8pwGSYU


      • bg-ope kirjoitti:

        *No milläs puut nostavat veden latvaan?*

        Vesimolekyylit muodostavat yhtenäisiä jonoja juurista lehtiin / neulasiin puun rungon vesiputkien, lehtisuonien ym. kautta. "Vesihissin" laittaa käyntiin aurinko, joka kuumentaa lehden pintaa ja saa veden haihtumaan ilmaan. Tämä saa aikaan imun jota tukee ohuissa vesiputkissa tapahtuva kapillaari-ilmiö: ilmaan haihtuva molekyyli kiskaisee edellistä mukanaan ylöspäin vesiputkessa. Ketjureaktio yltää aina juuren kärkeen saakka, jonne syntyy imu: puu saa imettyä vettä ja sen mukana ravinteita maasta.

        Kapillaari-ilmiö "vesihissi" selittävät myös aamukasteen.

        http://fi.m.wikipedia.org/wiki/Kapillaari-ilmiö

        *Ja millä puut laskevat nesteen juuriin syksyllä?*

        Painovoima hoitaa liikennettä alaspäin.

        Hississä...

        https://www.youtube.com/watch?v=_3ho8pwGSYU


      • qyfuyfquy kirjoitti:

        Evoluutio-uskovaisilla ei ole mitään tolkkua, on vain sokeaa uskonhurmiota.

        *Evoluutio-uskovaisilla ei ole mitään tolkkua, on vain sokeaa uskonhurmiota.*

        Lienee turha kysyä, mutta kysyn silti: Onko se että lapseni ei ole knoonini, uskonasia? Onko luonnonvalinta uskonasia? Onko geneettinen ajautuminen uskonasia? Ovatko rekombinaatio ja mutaatiot uskonasioita?

        Eivät ole. Ne ovat evoluution faktisia mekanismeja. Siksi jätän tuon harrastamanne perusteettoman jankkaamisen omaan arvoonsa.


      • Tuo kysymys näytti kyllä suuntautuvan neiti qyfuyfquylle, jos oikein tarkkaan katotaan hierarkiasta. Liekö joku viksumpi ihastunut Perähikiän tämänkertaiseen hienoon nikkiin.


      • RepeRuutikallo kirjoitti:

        Tuo kysymys näytti kyllä suuntautuvan neiti qyfuyfquylle, jos oikein tarkkaan katotaan hierarkiasta. Liekö joku viksumpi ihastunut Perähikiän tämänkertaiseen hienoon nikkiin.

        Jeps, niinpä olikin. Kiitos korjauksesta, poistan viestin.


      • Heh !
        makro = hoax kirjoitti:

        >

        Matematiikka on kaikkien luonnontieteiden perusta.

        >

        Voi voi, kun tässä on nyt vastakkain parhaaseen tietoon perustuva lukiokirja ja anonyymi suomi24 kirjoittaja. Todistappa että sellaista osoitusta ei siis ole.

        Matematiikka on kaikkien luonnontieteiden perusta. >>

        Mutta ei ole luonnontiede.


        Voi voi, kun tässä on nyt vastakkain parhaaseen tietoon perustuva lukiokirja ja anonyymi suomi24 kirjoittaja. >>

        Valitettavasti et pysty mitenkään osoittamaan, että kyseessä olisi parhaaseen mahdolliseen tietoon perustuva kirja (kysehän ei ole käsittääkseni edes lukiokirjasta vaan kertauskirjasta, jonka validius jo kyseenalaistettiin). Joudumme kuitenkin myöntämään, että esim. akkreditoidussa yliopistossa laajasti käytettävät oppikirjat ovat korkeatasoisempia. Niistä jokaisessa evoluutio myönnetään tieteelliseksi faktaksi ja esim. Maapallo miljardeja vuosia vanhaksi (joissa asiasta jotain sanotaan).

        Et siis oikein voi valita jotain kirjaa ja pitää sitä ehdottomana totuutena vain sen perusteella, että siellä esitetään uskontoasi tukeva todisteeton väite.

        Ja tosiaan, esität nyt tuossa, että USKONNON kirja olisi MATEMATIIKAN parasta tietoa.

        Menee siis naurettavaksi jälleen kerran tuo hölmöilysi.

        >

        Tieteessä (samoin kuin oikeudessa) positiivisen väitteen esittäjä on todistusvastuussa. Tämä tarkoittaa sitä, että jos väittää jonkun osoituksen olevan olemassa, väitteen esittäjän vastuulla on myös tämän osoituksen tuominen todisteeksi. Mikäli väitteen esittäjä ei pysty tätä tekemään, väite katsotaan huuhaaksi.

        [Jos et tätä ymmärrä, niin teen väitteen: saunatonttuja on olemassa. Jos et pysty todistamaan, ettei ole, Sinun on oman loogisen kuperkeikkasi perusteella uskoa saunatonttuihin tästä hetkestä eteenpäin.]

        Näin siis Sinä joudut esittämään tuon todisteen tai jos jää esittämättä, tuokin väite lisätään kreationistien valheiden jo ennestään loputtoman pitkään listaan.

        Ja sitten kun ihan aidosti mietitään niin, mikä olisi todennäköisyys sille, että esim. tällä palstalla KUKAAN ei olisi koskaan kuullutkaan kaiken nykytietämyksen murskaavasta tieteellisestä todistuksesta. Niin, olisihan tuo esitetty täällä jo tuhanteen kertaan ja suljettu evoluutiota kannattavien suut yksiselitteisesti, mutta kun sellaista todistetta ei ole olemassa - ei varmasti oikeasti edes hihhulien omissa kuvitelmissa. Järki käteen nyt vaikka mielikuvitusukkeleihin uskotkin kuin pikkulapsi joulupukkiin.

        [Tieteellinen lähde olisi sitä parasta tietoa, johon väitteensä ylipäätään voisi perustaa.]


    • Evoluutioteorialta puuttuu se tieteellinen todiste eli mekanismi joka sen mahdollistaisi.

      Tässä kokeellisen luonnontieteen expertti mikrobi ja entsyymiasiantuntija bioprosessitekniikan emeritusprofessori Matti Leisola kertoo haastattelussani ettei evoluutiolle mikrobeista ihmiseksi ole kokeellisten havaintojen mukaan mekanismia:

      http://www.youtube.com/watch?v=0dzKMlQeCQ8

      • "...ettei evoluutiolle mikrobeista ihmiseksi ole kokeellisten havaintojen mukaan mekanismia"

        Aikaa muinaisesta mikrobista nykyiseen ihmiseen on kulunut yli 3 miljardia vuotta. Mekanismi on se hyvin tunnettu muuntelu & luonnonvalinta. Kokeellisia havaintoja on vasta yli sadalta vuodelta, mutta jos eliöiden perimiä tarkastellaan, niin sieltähän niitä havaintoja tapahtumista on saatavilla ihan tässäkin ajassa.

        Leisola vaan puhuu soopaa.


      • antimytomaani_orig kirjoitti:

        "...ettei evoluutiolle mikrobeista ihmiseksi ole kokeellisten havaintojen mukaan mekanismia"

        Aikaa muinaisesta mikrobista nykyiseen ihmiseen on kulunut yli 3 miljardia vuotta. Mekanismi on se hyvin tunnettu muuntelu & luonnonvalinta. Kokeellisia havaintoja on vasta yli sadalta vuodelta, mutta jos eliöiden perimiä tarkastellaan, niin sieltähän niitä havaintoja tapahtumista on saatavilla ihan tässäkin ajassa.

        Leisola vaan puhuu soopaa.

        Leisola ja Torppa tarkoittavat tietenkin sitä, ettei laboratoriossa ole pystytty kehittämään mikrobista ihmistä. :-)


      • Ota selvää tieteilijöiden yli 99 prosentin käsityksistä äläkä koko ajan vetoa mikroskooppiseen vähemmistöön, vaikka kuinka olisi kaveristasi kyse.

        Harvalukuisia haahuilevia yksityisajattelijoita löytyy kaikilta aloilta ja myös professoreista. Biologiasta ei ilmeisesti kuitenkaan yhtäkään? Jos löytyisi, olisit varmaan jo lainannut...


      • RepeRuutikallo kirjoitti:

        Ota selvää tieteilijöiden yli 99 prosentin käsityksistä äläkä koko ajan vetoa mikroskooppiseen vähemmistöön, vaikka kuinka olisi kaveristasi kyse.

        Harvalukuisia haahuilevia yksityisajattelijoita löytyy kaikilta aloilta ja myös professoreista. Biologiasta ei ilmeisesti kuitenkaan yhtäkään? Jos löytyisi, olisit varmaan jo lainannut...

        Tämä 99 % on sinun itse keksimä uskomus - todellisuus on jotain täysin muuta.

        Bioprosessitekniikan emeritusprofessori Matti Leisola kirjoittaa kirjassaan evoluutiouskon ihmemaassa s,36 seuraavaa:

        - "Olen yli neljänkymmenen vuoden ajan käynyt elämän ja lajien syntyä koskevaa keskustelua tiedeyhteisön sisällä ja sen ulkopuolella. Lähes kaikki tuntemani sadat tutkijat myöntävät kahdenkeskisissä, luottamuksellisissa keskusteluissa, ettei tieteellä ole selitystä geneettisen kielen, proteiinien, solukalvojen, aineenvaihduntareaktioiden, säätelyjärjestelmien, solujen ja biologian perusrakenteiden alkuperälle."


      • *Evoluutioteorialta puuttuu se tieteellinen todiste eli mekanismi joka sen mahdollistaisi.*

        Älä valehtele. Evoluution mekanismeja ovat mm. luonnonvalinta ja geneettinen ajautuminen, jotka ovat luonnossa esiintyviä todeksi havaittuja ilmiöitä. Myös geneettinen muuntelu on fakta, jota sinä valehtelullasi et saa häviämään.

        Seuraavaksi yrität sitten valehdella tietäväsi biologeja paremmin mikä on evoluutiota ja mikä ei, ja miten laji määritellään.

        Mitäpä jos yrittäisit vaihteeksi mennä fyysikoille selventämään, kuinka tiedät heitä paremmin mikä sateenkaaren aiheuttaa. Ja mikä sinun mielestäsi on optiikkaa ja mikä ei.


      • t-torppa kirjoitti:

        Tämä 99 % on sinun itse keksimä uskomus - todellisuus on jotain täysin muuta.

        Bioprosessitekniikan emeritusprofessori Matti Leisola kirjoittaa kirjassaan evoluutiouskon ihmemaassa s,36 seuraavaa:

        - "Olen yli neljänkymmenen vuoden ajan käynyt elämän ja lajien syntyä koskevaa keskustelua tiedeyhteisön sisällä ja sen ulkopuolella. Lähes kaikki tuntemani sadat tutkijat myöntävät kahdenkeskisissä, luottamuksellisissa keskusteluissa, ettei tieteellä ole selitystä geneettisen kielen, proteiinien, solukalvojen, aineenvaihduntareaktioiden, säätelyjärjestelmien, solujen ja biologian perusrakenteiden alkuperälle."

        "... tieteellä ole selitystä geneettisen kielen, proteiinien, solukalvojen, aineenvaihduntareaktioiden, säätelyjärjestelmien, solujen ja biologian perusrakenteiden alkuperälle."

        Mikä faktisesti voi muuten pitääkin paikkansa. Ei ole olemassa varmaa tietoa, hypoteeseja kyllä, samoin hypoteesien ympäriltä uutta tutkimustakin.

        Tiedon puute ei ole kuitenkaan tarkoita, että selityksenä olisi automaattisesti "Jumala loi" tai joku muu yliluonnolliseen syyhyn perustuva väite.

        Tuo ei kuitenkaan anna mitään perustetta väittää, ettei muka tiedettäisi evoluutiomekanismeja (koska niistä on nimittäin havaintoja ja niiden pohjalta on jopa tehty kokeita!).


      • Råger Mer
        bg-ope kirjoitti:

        *Evoluutioteorialta puuttuu se tieteellinen todiste eli mekanismi joka sen mahdollistaisi.*

        Älä valehtele. Evoluution mekanismeja ovat mm. luonnonvalinta ja geneettinen ajautuminen, jotka ovat luonnossa esiintyviä todeksi havaittuja ilmiöitä. Myös geneettinen muuntelu on fakta, jota sinä valehtelullasi et saa häviämään.

        Seuraavaksi yrität sitten valehdella tietäväsi biologeja paremmin mikä on evoluutiota ja mikä ei, ja miten laji määritellään.

        Mitäpä jos yrittäisit vaihteeksi mennä fyysikoille selventämään, kuinka tiedät heitä paremmin mikä sateenkaaren aiheuttaa. Ja mikä sinun mielestäsi on optiikkaa ja mikä ei.

        Tuhkimo-satu on täysin evoluutioteorian puolella. Siinäkin hiiret muuttuvat hevosiksi yliluonnollisen taian avulla.


      • t-torppa kirjoitti:

        Tämä 99 % on sinun itse keksimä uskomus - todellisuus on jotain täysin muuta.

        Bioprosessitekniikan emeritusprofessori Matti Leisola kirjoittaa kirjassaan evoluutiouskon ihmemaassa s,36 seuraavaa:

        - "Olen yli neljänkymmenen vuoden ajan käynyt elämän ja lajien syntyä koskevaa keskustelua tiedeyhteisön sisällä ja sen ulkopuolella. Lähes kaikki tuntemani sadat tutkijat myöntävät kahdenkeskisissä, luottamuksellisissa keskusteluissa, ettei tieteellä ole selitystä geneettisen kielen, proteiinien, solukalvojen, aineenvaihduntareaktioiden, säätelyjärjestelmien, solujen ja biologian perusrakenteiden alkuperälle."

        Sinuako ei yhtään häiritse, että kamusi levittelee "luottamuksellisia keskusteluja" ympäriinsä ja sinä hänen narrinaan perässä?

        Toisaalta, koska Leisola on todistetusti jäänyt monet kerrat apologeettisista valheistaan kiinni, ei ikävä kyllä ole mitään syytä uskoa hänen puhuvan totta tässäkään.

        Tosiasia on, että uransa tyhmyyttään sössineen Lennart Saaren lisäksi et pysty löytämään Suomesta tai edes pohjoismaista yhtäkään elävää yliopistobiologia, joka edes vapaa-aikanaan ja aseella uhattuna allekirjoittaisi kreationismista sanaakaan. Hyvä jos koko EU:sta löydät. Yhdysvalloista saattaisi löytyä, mutta ilmeisesti et löydä, kun et heitäkään lainaa.

        Hohhoijaa.


      • Råger Mer kirjoitti:

        Tuhkimo-satu on täysin evoluutioteorian puolella. Siinäkin hiiret muuttuvat hevosiksi yliluonnollisen taian avulla.

        Kuules nyt satusetä (tai -täti), ei evoluutioteoriassa ole mitään yliluonnollisia taikoja esitettynä.

        Jos ole eri mieltä, kerropa esimerkki evoluutioteorian siitä osasta, jossa olisi sinun mielestäsi jotain yliluonnollista.


      • antimytomaani_orig kirjoitti:

        "... tieteellä ole selitystä geneettisen kielen, proteiinien, solukalvojen, aineenvaihduntareaktioiden, säätelyjärjestelmien, solujen ja biologian perusrakenteiden alkuperälle."

        Mikä faktisesti voi muuten pitääkin paikkansa. Ei ole olemassa varmaa tietoa, hypoteeseja kyllä, samoin hypoteesien ympäriltä uutta tutkimustakin.

        Tiedon puute ei ole kuitenkaan tarkoita, että selityksenä olisi automaattisesti "Jumala loi" tai joku muu yliluonnolliseen syyhyn perustuva väite.

        Tuo ei kuitenkaan anna mitään perustetta väittää, ettei muka tiedettäisi evoluutiomekanismeja (koska niistä on nimittäin havaintoja ja niiden pohjalta on jopa tehty kokeita!).

        Se että tieteellä ei ole selitystä biologian perusrakenteiden alkuperälle tarkoittaa sitä ettei tiede nimenomaan tunne mekanismia joka ne selittäisi.

        Biologian perusrakenteita voidaan muunnella niissä variaatio rajoissa johon perimän informaatio kapasiteetti riittää - geenivaraston muuntelu kapasiteetti on kuitenkin rajallinen eikä näitä rajoja voi ylittää - ilman että olisi mekanismi jossa uutta tietoa luotaisiin tyhjästä..

        Kaiken lisäksi jo olemassa olevien rakenteiden muuntelu ei kerro mitään niiden alkuperästä. Evoluutioteorialle olisi tärkeää vain tietää mekanismi biologisten rakenteiden alkuperälle mutta sitä ei nimenomaan tunneta ja tämän vuoksi koko teoria on vain suuri uskomusjärjestelmä.


      • RepeRuutikallo kirjoitti:

        Sinuako ei yhtään häiritse, että kamusi levittelee "luottamuksellisia keskusteluja" ympäriinsä ja sinä hänen narrinaan perässä?

        Toisaalta, koska Leisola on todistetusti jäänyt monet kerrat apologeettisista valheistaan kiinni, ei ikävä kyllä ole mitään syytä uskoa hänen puhuvan totta tässäkään.

        Tosiasia on, että uransa tyhmyyttään sössineen Lennart Saaren lisäksi et pysty löytämään Suomesta tai edes pohjoismaista yhtäkään elävää yliopistobiologia, joka edes vapaa-aikanaan ja aseella uhattuna allekirjoittaisi kreationismista sanaakaan. Hyvä jos koko EU:sta löydät. Yhdysvalloista saattaisi löytyä, mutta ilmeisesti et löydä, kun et heitäkään lainaa.

        Hohhoijaa.

        Ei Matti paljastanut henkilöitä koska keskustelut olivat luottamuksellisia. Kyse ei ollut kreationismin tukemisesta vaan siitä ettei nämä tieteilijät tunne prosessia joka mahdollistaisi evoluution mikrobeista ihmiseksi.


      • bg-ope kirjoitti:

        *Evoluutioteorialta puuttuu se tieteellinen todiste eli mekanismi joka sen mahdollistaisi.*

        Älä valehtele. Evoluution mekanismeja ovat mm. luonnonvalinta ja geneettinen ajautuminen, jotka ovat luonnossa esiintyviä todeksi havaittuja ilmiöitä. Myös geneettinen muuntelu on fakta, jota sinä valehtelullasi et saa häviämään.

        Seuraavaksi yrität sitten valehdella tietäväsi biologeja paremmin mikä on evoluutiota ja mikä ei, ja miten laji määritellään.

        Mitäpä jos yrittäisit vaihteeksi mennä fyysikoille selventämään, kuinka tiedät heitä paremmin mikä sateenkaaren aiheuttaa. Ja mikä sinun mielestäsi on optiikkaa ja mikä ei.

        MUTAATIOT

        Kasvigeneetikko Dr John Sanford kertoo mutaatioista seuraavaa;

        - "Perimät ovat ne perinnölliset tekniset tiedot, jotka mahdollistavat elämän olemassaolon. Tekniset tiedot ovat selvästi luonnostaan tarkoin määrätty. Tämä tarkoittaa, että satunnaiset muutokset teknisissä tiedoissa sekoittavat tietoa erittäin suurella varmuudella."

        - "Mutaatiot ovat tekstinkäsittelyvirheitä solun käyttöohjeissa. Mutaatiot tuhoavat systemaattisesti geneettistä informaatiota – aivan kuten tekstinkäsittelyvirheet tuhoavat kirjoitettua informaatiota.. Mitä enemmän mutaatioita sitä vähemmän informaatiota. Tämä on mutaatioprosessin perusperiaate."

        Mutaatiot tuhoavat systemaattisesti geneettistä informaatiota - sillä ne ovat tekstinkäsittelyvirheitä solun käyttöohjeissa. Mutaatiot tuhoavat solun ohjeiden merkityksellisyyttä - koska satunnaiset muutokset teknisissä tiedoissa sekoittavat tietoa erittäin suurella varmuudella. Tässä valossa mutaatioita voisi verrata autoon kertyvään ruosteeseen. Autoon kertyy ajansaatossa ruostetta joka rappeuttaa - tuhoaa auton merkityksellisiä rakenteita. Mitä ruosteisempi auto sitä vähemmän on toiminnallista merkityksellisyyttä autossa. Mitä vanhempi auto sitä ruosteisempi. Ruosteella ei ole älyä eikä kykyä kirjoittaa auton rakennus ja toimintaohjeita samoin kun kirjoitusvirheet eivät kykene kirjoittamaan tietosanakirjaa.



        LUONNONVALINTA

        Dr John Sanford kertoo;

        - "Valinta auttaa kyllä. Se poistaa joukosta pahimmat mutaatiot. Tämä hidastaa mutaatioiden aiheuttamaa rappeutumista.

        Lisäksi, hyvin harvoin syntyy hyödyllinen mutaatio, jolla on tarpeeksi iso vaikutus tullakseen valituksi – josta seuraisi jokin sopeutunut muunnos tai jonkin asteista hienosäätöä. Tämä auttaa myös rappeutumisen hidastamisessa. Valinta kuitenkin karsii huonoista mutaatioista vain hyvin pienen murto-osan. Ylivoimainen enemmistö huonoista mutaatioista on sellaisia, joita kertyy jatkuvasti, ne ovat aivan liian hiuksenhienoja – niiden vaikutus on aivan liian pieni – jotta valinnalla olisi vaikutusta niihin. Kääntöpuolella taas lähes kaikki hyödylliset mutaatiot (siinä määrin, kuin niitä ilmaantuu) ovat riippumattomia valintaprosessista, koska ne poikkeuksetta aiheuttavat biologiseen toiminnallisuuteen vain pikkuriikkisiä lisäyksiä.

        Valinta hidastaa mutaatioista johtuvaa rappeutumista, mutta se ei itse asiassa kykene edes aloittamaan rappeutumisen pysäyttämistä. Joten jopa kiivaassa valinnassa, evoluutio kulkee väärään suuntaan – kohti sukupuuttoa!"

        Valinta pystyy vain hidastamaan mutaatioista johtuvaa rappeutumista - mutta ei kykene edes pysäyttämään sitä. Näin siksi että suurin osa mutaatioista ovat vain vähän haitallisia ja tämän vuoksi valinta ei näe niitä. Valinta näkee vain pahimmat mutaatiot ja tämän vuoksi vähän haitalliset mutaatiot ovat kuin ruoste autossa jota silmä ei aluksi näe mutta ajansaatossa niitä kertyy siinä määrin että ne aiheuttavat toiminnallisia häiriöitä.

        Kyseiset hyödylliset mutaatiot joita on vain murto-osa verrattuna haitallisiin ovat myös virheitä jossa informaatiota menetetään mutta näistä virheistä voi olla joissain oloissa hyötyä - siksi niitä kutsutaan hyödyllisiksi mutaatioiksi. Ne eivät kehitä uutta informaatiota vaan tuhoavat jo olemassa olevaa mutta joskus jo olemassa olevan informaation tuhoutuminen voi tuoda valintaedun.

        Esimerkiksi kun koppikselta tuhoutuu siipien rakennusohjeet mutaation seurauksena niin se ei lennä tuulisissa oloissa mereen ja huku. Näin siivet tuhonnut mutaatio voi olla hyödyllinen tietyissä (tuulisissa) oloissa mutta kyseessä oli tapahtuma jossa geneettistä merkityksellistä informaatiota menetettiin. Tällaisilla 'hyödyllisillä' mutaatioilla ei voida perustella mikrobista - ihmiseksi kehitystä sillä kyseinen kehitys tarvitsisi autokuormittain ennestään täysin uusia merkityksellisiä rakennus ja toimintaohjeita (geneettistä informaatiota).

        Auton kohdalla hyödyllinen mutaatio olosuhteisiin nähden olisi esimerkiksi sellainen tapahtuma jossa autosta otetaan takaspoileri pois jotta se kulkee suoralla lujempaa. Spoilerin poisto tuo tiettyihin olosuhteisiin etua mutta kyseessä oli jo olemassa olevan ominaisuuden hävittäminen ei uuden kehittäminen. Jo olemassa olevien rakenteiden hävittäminen on vastakohta sille mitä evoluutioteorian opettama kehitys tarvitsisi toimiakseensa.


      • t-torppa kirjoitti:

        Tämä 99 % on sinun itse keksimä uskomus - todellisuus on jotain täysin muuta.

        Bioprosessitekniikan emeritusprofessori Matti Leisola kirjoittaa kirjassaan evoluutiouskon ihmemaassa s,36 seuraavaa:

        - "Olen yli neljänkymmenen vuoden ajan käynyt elämän ja lajien syntyä koskevaa keskustelua tiedeyhteisön sisällä ja sen ulkopuolella. Lähes kaikki tuntemani sadat tutkijat myöntävät kahdenkeskisissä, luottamuksellisissa keskusteluissa, ettei tieteellä ole selitystä geneettisen kielen, proteiinien, solukalvojen, aineenvaihduntareaktioiden, säätelyjärjestelmien, solujen ja biologian perusrakenteiden alkuperälle."

        Ei hyvää päivää tuota logiikkaa: kreationistit Torppa ja hänen palvomansa Leisola hihkuvat ajatuksella, että jos tiede ei osaa selittää jotain niin...JEEEEESUS!

        Ei ole ilmeisesti kumpaisellakaan käynyt missään vaiheessa mielessä että a) tiede ei osaa selittää kaikkea b) tiede osaa selittää vuosi vuodelta yhä enemmän asioita, mutta uskonnot eivätkä niiden pyhät kirjoitukset edelleenkään ole tuoneet mitään uutta tietoa maailmankaikkeudesta.

        Vielä kohta c): Tiede ei todennäkösesti koskaan pysty selittämään ihan kaikkea maailmankaikkeudessa, ja hyvä niin: silloin on jotain tutkittavaa ja selitettävää. Ymmärtämättömyys / tietämättömyys ei ole todiste mistään muusta kuin että ymmärtämätön ei ymmärrä tai tiedä.

        Tieteen tehtävä on pyörittää uutta tietoa tuovaa ratasta, jonka tielle uskonnot ja uskovaiset ovat yrittäneet tuoda mitä eriskummallisimpia esteitä. Jyrkän islaminuskon vaikutus tieteen tekemiseen arabimaissa on tästä hyvä esimerkki. Onneksi Suomessa ei olla noin syvällä uskonsuossa, vaan olemme sekulaari valtio. Täällä eivät Leisolan ja Torpan kaltaiset fundamentalistit mahda sivistyksen etenemiselle mitään.


      • t-torppa kirjoitti:

        Se että tieteellä ei ole selitystä biologian perusrakenteiden alkuperälle tarkoittaa sitä ettei tiede nimenomaan tunne mekanismia joka ne selittäisi.

        Biologian perusrakenteita voidaan muunnella niissä variaatio rajoissa johon perimän informaatio kapasiteetti riittää - geenivaraston muuntelu kapasiteetti on kuitenkin rajallinen eikä näitä rajoja voi ylittää - ilman että olisi mekanismi jossa uutta tietoa luotaisiin tyhjästä..

        Kaiken lisäksi jo olemassa olevien rakenteiden muuntelu ei kerro mitään niiden alkuperästä. Evoluutioteorialle olisi tärkeää vain tietää mekanismi biologisten rakenteiden alkuperälle mutta sitä ei nimenomaan tunneta ja tämän vuoksi koko teoria on vain suuri uskomusjärjestelmä.

        >Evoluutioteorialle olisi tärkeää vain tietää mekanismi biologisten rakenteiden alkuperälle mutta sitä ei nimenomaan tunneta ja tämän vuoksi koko teoria on vain suuri uskomusjärjestelmä.

        Tämän typeryyden inttäminen on näköjään keskeinen osa omaa uskomusjärjestelmääsi. Ei se siitä silti todeksi muutu.


      • bg-ope kirjoitti:

        Ei hyvää päivää tuota logiikkaa: kreationistit Torppa ja hänen palvomansa Leisola hihkuvat ajatuksella, että jos tiede ei osaa selittää jotain niin...JEEEEESUS!

        Ei ole ilmeisesti kumpaisellakaan käynyt missään vaiheessa mielessä että a) tiede ei osaa selittää kaikkea b) tiede osaa selittää vuosi vuodelta yhä enemmän asioita, mutta uskonnot eivätkä niiden pyhät kirjoitukset edelleenkään ole tuoneet mitään uutta tietoa maailmankaikkeudesta.

        Vielä kohta c): Tiede ei todennäkösesti koskaan pysty selittämään ihan kaikkea maailmankaikkeudessa, ja hyvä niin: silloin on jotain tutkittavaa ja selitettävää. Ymmärtämättömyys / tietämättömyys ei ole todiste mistään muusta kuin että ymmärtämätön ei ymmärrä tai tiedä.

        Tieteen tehtävä on pyörittää uutta tietoa tuovaa ratasta, jonka tielle uskonnot ja uskovaiset ovat yrittäneet tuoda mitä eriskummallisimpia esteitä. Jyrkän islaminuskon vaikutus tieteen tekemiseen arabimaissa on tästä hyvä esimerkki. Onneksi Suomessa ei olla noin syvällä uskonsuossa, vaan olemme sekulaari valtio. Täällä eivät Leisolan ja Torpan kaltaiset fundamentalistit mahda sivistyksen etenemiselle mitään.

        Olisi edelleen kiinnostavaa tietää valehteleeko Leisola tahallaan vai vastoin parempaa tietoa.


      • Råger Mer kirjoitti:

        Tuhkimo-satu on täysin evoluutioteorian puolella. Siinäkin hiiret muuttuvat hevosiksi yliluonnollisen taian avulla.

        Tuhkimonpalvoja on näköjään jalostunut nikkivarkaaksikin. Söpöä (vai mikä se sana nyt oli).

        Kaikella tässä tapauksessa mahdollisella kunnioituksella: vitsisi ei enää naurata.


      • t-torppa kirjoitti:

        Ei Matti paljastanut henkilöitä koska keskustelut olivat luottamuksellisia. Kyse ei ollut kreationismin tukemisesta vaan siitä ettei nämä tieteilijät tunne prosessia joka mahdollistaisi evoluution mikrobeista ihmiseksi.

        >Kyse ei ollut kreationismin tukemisesta vaan siitä ettei nämä tieteilijät tunne prosessia joka mahdollistaisi evoluution mikrobeista ihmiseksi.

        No no. Mitäs sävellystä tämä nyt on olevinaan? Mitään tuollaistahan Leisola ei väittänyt kollegojensa sanoneen. Tai sitten et vain vieläkään tajua näistä asioista yhtään mitään ja kaikki on sikäli sulle yhtä mössöä.

        Joka tapauksessa jäit taas valheesta kiinni; toivottavasti omatuntoon koskee edes vähän, vaikka pelkäänkin että eipä tunnu missään.


      • salmiakkisaluki kirjoitti:

        Olisi edelleen kiinnostavaa tietää valehteleeko Leisola tahallaan vai vastoin parempaa tietoa.

        Olettaisin että hänestäkään apologeettinen valehtelu ei ole varsinaista valehtelua, vaan itse asiassa eräänlainen todenpuhumisen ylväin aste, kun se raamattu kuitenkin on oikeassa.


      • t-torppa kirjoitti:

        MUTAATIOT

        Kasvigeneetikko Dr John Sanford kertoo mutaatioista seuraavaa;

        - "Perimät ovat ne perinnölliset tekniset tiedot, jotka mahdollistavat elämän olemassaolon. Tekniset tiedot ovat selvästi luonnostaan tarkoin määrätty. Tämä tarkoittaa, että satunnaiset muutokset teknisissä tiedoissa sekoittavat tietoa erittäin suurella varmuudella."

        - "Mutaatiot ovat tekstinkäsittelyvirheitä solun käyttöohjeissa. Mutaatiot tuhoavat systemaattisesti geneettistä informaatiota – aivan kuten tekstinkäsittelyvirheet tuhoavat kirjoitettua informaatiota.. Mitä enemmän mutaatioita sitä vähemmän informaatiota. Tämä on mutaatioprosessin perusperiaate."

        Mutaatiot tuhoavat systemaattisesti geneettistä informaatiota - sillä ne ovat tekstinkäsittelyvirheitä solun käyttöohjeissa. Mutaatiot tuhoavat solun ohjeiden merkityksellisyyttä - koska satunnaiset muutokset teknisissä tiedoissa sekoittavat tietoa erittäin suurella varmuudella. Tässä valossa mutaatioita voisi verrata autoon kertyvään ruosteeseen. Autoon kertyy ajansaatossa ruostetta joka rappeuttaa - tuhoaa auton merkityksellisiä rakenteita. Mitä ruosteisempi auto sitä vähemmän on toiminnallista merkityksellisyyttä autossa. Mitä vanhempi auto sitä ruosteisempi. Ruosteella ei ole älyä eikä kykyä kirjoittaa auton rakennus ja toimintaohjeita samoin kun kirjoitusvirheet eivät kykene kirjoittamaan tietosanakirjaa.



        LUONNONVALINTA

        Dr John Sanford kertoo;

        - "Valinta auttaa kyllä. Se poistaa joukosta pahimmat mutaatiot. Tämä hidastaa mutaatioiden aiheuttamaa rappeutumista.

        Lisäksi, hyvin harvoin syntyy hyödyllinen mutaatio, jolla on tarpeeksi iso vaikutus tullakseen valituksi – josta seuraisi jokin sopeutunut muunnos tai jonkin asteista hienosäätöä. Tämä auttaa myös rappeutumisen hidastamisessa. Valinta kuitenkin karsii huonoista mutaatioista vain hyvin pienen murto-osan. Ylivoimainen enemmistö huonoista mutaatioista on sellaisia, joita kertyy jatkuvasti, ne ovat aivan liian hiuksenhienoja – niiden vaikutus on aivan liian pieni – jotta valinnalla olisi vaikutusta niihin. Kääntöpuolella taas lähes kaikki hyödylliset mutaatiot (siinä määrin, kuin niitä ilmaantuu) ovat riippumattomia valintaprosessista, koska ne poikkeuksetta aiheuttavat biologiseen toiminnallisuuteen vain pikkuriikkisiä lisäyksiä.

        Valinta hidastaa mutaatioista johtuvaa rappeutumista, mutta se ei itse asiassa kykene edes aloittamaan rappeutumisen pysäyttämistä. Joten jopa kiivaassa valinnassa, evoluutio kulkee väärään suuntaan – kohti sukupuuttoa!"

        Valinta pystyy vain hidastamaan mutaatioista johtuvaa rappeutumista - mutta ei kykene edes pysäyttämään sitä. Näin siksi että suurin osa mutaatioista ovat vain vähän haitallisia ja tämän vuoksi valinta ei näe niitä. Valinta näkee vain pahimmat mutaatiot ja tämän vuoksi vähän haitalliset mutaatiot ovat kuin ruoste autossa jota silmä ei aluksi näe mutta ajansaatossa niitä kertyy siinä määrin että ne aiheuttavat toiminnallisia häiriöitä.

        Kyseiset hyödylliset mutaatiot joita on vain murto-osa verrattuna haitallisiin ovat myös virheitä jossa informaatiota menetetään mutta näistä virheistä voi olla joissain oloissa hyötyä - siksi niitä kutsutaan hyödyllisiksi mutaatioiksi. Ne eivät kehitä uutta informaatiota vaan tuhoavat jo olemassa olevaa mutta joskus jo olemassa olevan informaation tuhoutuminen voi tuoda valintaedun.

        Esimerkiksi kun koppikselta tuhoutuu siipien rakennusohjeet mutaation seurauksena niin se ei lennä tuulisissa oloissa mereen ja huku. Näin siivet tuhonnut mutaatio voi olla hyödyllinen tietyissä (tuulisissa) oloissa mutta kyseessä oli tapahtuma jossa geneettistä merkityksellistä informaatiota menetettiin. Tällaisilla 'hyödyllisillä' mutaatioilla ei voida perustella mikrobista - ihmiseksi kehitystä sillä kyseinen kehitys tarvitsisi autokuormittain ennestään täysin uusia merkityksellisiä rakennus ja toimintaohjeita (geneettistä informaatiota).

        Auton kohdalla hyödyllinen mutaatio olosuhteisiin nähden olisi esimerkiksi sellainen tapahtuma jossa autosta otetaan takaspoileri pois jotta se kulkee suoralla lujempaa. Spoilerin poisto tuo tiettyihin olosuhteisiin etua mutta kyseessä oli jo olemassa olevan ominaisuuden hävittäminen ei uuden kehittäminen. Jo olemassa olevien rakenteiden hävittäminen on vastakohta sille mitä evoluutioteorian opettama kehitys tarvitsisi toimiakseensa.

        Lainaapa vaihteeksi sitä geneetikkojen yli 99:ää prosenttia, joka ei allekirjoita kreationismin tueksi keksityistä "perusteluista" sanaakaan.
        Heidän kirjallista tuotantoaan on olemassa hyllymetreittäin ja lisää syntyy vuosittain, vaikka ethän sä siitä mitään tajua.


      • t-torppa kirjoitti:

        MUTAATIOT

        Kasvigeneetikko Dr John Sanford kertoo mutaatioista seuraavaa;

        - "Perimät ovat ne perinnölliset tekniset tiedot, jotka mahdollistavat elämän olemassaolon. Tekniset tiedot ovat selvästi luonnostaan tarkoin määrätty. Tämä tarkoittaa, että satunnaiset muutokset teknisissä tiedoissa sekoittavat tietoa erittäin suurella varmuudella."

        - "Mutaatiot ovat tekstinkäsittelyvirheitä solun käyttöohjeissa. Mutaatiot tuhoavat systemaattisesti geneettistä informaatiota – aivan kuten tekstinkäsittelyvirheet tuhoavat kirjoitettua informaatiota.. Mitä enemmän mutaatioita sitä vähemmän informaatiota. Tämä on mutaatioprosessin perusperiaate."

        Mutaatiot tuhoavat systemaattisesti geneettistä informaatiota - sillä ne ovat tekstinkäsittelyvirheitä solun käyttöohjeissa. Mutaatiot tuhoavat solun ohjeiden merkityksellisyyttä - koska satunnaiset muutokset teknisissä tiedoissa sekoittavat tietoa erittäin suurella varmuudella. Tässä valossa mutaatioita voisi verrata autoon kertyvään ruosteeseen. Autoon kertyy ajansaatossa ruostetta joka rappeuttaa - tuhoaa auton merkityksellisiä rakenteita. Mitä ruosteisempi auto sitä vähemmän on toiminnallista merkityksellisyyttä autossa. Mitä vanhempi auto sitä ruosteisempi. Ruosteella ei ole älyä eikä kykyä kirjoittaa auton rakennus ja toimintaohjeita samoin kun kirjoitusvirheet eivät kykene kirjoittamaan tietosanakirjaa.



        LUONNONVALINTA

        Dr John Sanford kertoo;

        - "Valinta auttaa kyllä. Se poistaa joukosta pahimmat mutaatiot. Tämä hidastaa mutaatioiden aiheuttamaa rappeutumista.

        Lisäksi, hyvin harvoin syntyy hyödyllinen mutaatio, jolla on tarpeeksi iso vaikutus tullakseen valituksi – josta seuraisi jokin sopeutunut muunnos tai jonkin asteista hienosäätöä. Tämä auttaa myös rappeutumisen hidastamisessa. Valinta kuitenkin karsii huonoista mutaatioista vain hyvin pienen murto-osan. Ylivoimainen enemmistö huonoista mutaatioista on sellaisia, joita kertyy jatkuvasti, ne ovat aivan liian hiuksenhienoja – niiden vaikutus on aivan liian pieni – jotta valinnalla olisi vaikutusta niihin. Kääntöpuolella taas lähes kaikki hyödylliset mutaatiot (siinä määrin, kuin niitä ilmaantuu) ovat riippumattomia valintaprosessista, koska ne poikkeuksetta aiheuttavat biologiseen toiminnallisuuteen vain pikkuriikkisiä lisäyksiä.

        Valinta hidastaa mutaatioista johtuvaa rappeutumista, mutta se ei itse asiassa kykene edes aloittamaan rappeutumisen pysäyttämistä. Joten jopa kiivaassa valinnassa, evoluutio kulkee väärään suuntaan – kohti sukupuuttoa!"

        Valinta pystyy vain hidastamaan mutaatioista johtuvaa rappeutumista - mutta ei kykene edes pysäyttämään sitä. Näin siksi että suurin osa mutaatioista ovat vain vähän haitallisia ja tämän vuoksi valinta ei näe niitä. Valinta näkee vain pahimmat mutaatiot ja tämän vuoksi vähän haitalliset mutaatiot ovat kuin ruoste autossa jota silmä ei aluksi näe mutta ajansaatossa niitä kertyy siinä määrin että ne aiheuttavat toiminnallisia häiriöitä.

        Kyseiset hyödylliset mutaatiot joita on vain murto-osa verrattuna haitallisiin ovat myös virheitä jossa informaatiota menetetään mutta näistä virheistä voi olla joissain oloissa hyötyä - siksi niitä kutsutaan hyödyllisiksi mutaatioiksi. Ne eivät kehitä uutta informaatiota vaan tuhoavat jo olemassa olevaa mutta joskus jo olemassa olevan informaation tuhoutuminen voi tuoda valintaedun.

        Esimerkiksi kun koppikselta tuhoutuu siipien rakennusohjeet mutaation seurauksena niin se ei lennä tuulisissa oloissa mereen ja huku. Näin siivet tuhonnut mutaatio voi olla hyödyllinen tietyissä (tuulisissa) oloissa mutta kyseessä oli tapahtuma jossa geneettistä merkityksellistä informaatiota menetettiin. Tällaisilla 'hyödyllisillä' mutaatioilla ei voida perustella mikrobista - ihmiseksi kehitystä sillä kyseinen kehitys tarvitsisi autokuormittain ennestään täysin uusia merkityksellisiä rakennus ja toimintaohjeita (geneettistä informaatiota).

        Auton kohdalla hyödyllinen mutaatio olosuhteisiin nähden olisi esimerkiksi sellainen tapahtuma jossa autosta otetaan takaspoileri pois jotta se kulkee suoralla lujempaa. Spoilerin poisto tuo tiettyihin olosuhteisiin etua mutta kyseessä oli jo olemassa olevan ominaisuuden hävittäminen ei uuden kehittäminen. Jo olemassa olevien rakenteiden hävittäminen on vastakohta sille mitä evoluutioteorian opettama kehitys tarvitsisi toimiakseensa.

        Jos Sanford jatkaa johdonmukaisesti linjallaan - "Formerly an atheist[10] since the mid-1980s, Sanford has looked into theistic evolution (1985–late 1990s), Old Earth creationism (late 1990s), and Young Earth creationism (2000–present)." - hän edennee JC:n kaltaiseksi megalomaanikoksi ja tee-se-itse-jumalaksi. Saisit hävetä Toni kun palvot epäjumalinasi Leisolan ja Sanfordin kaltaisia.

        http://en.wikipedia.org/wiki/John_C._Sanford


      • t-torppa kirjoitti:

        Se että tieteellä ei ole selitystä biologian perusrakenteiden alkuperälle tarkoittaa sitä ettei tiede nimenomaan tunne mekanismia joka ne selittäisi.

        Biologian perusrakenteita voidaan muunnella niissä variaatio rajoissa johon perimän informaatio kapasiteetti riittää - geenivaraston muuntelu kapasiteetti on kuitenkin rajallinen eikä näitä rajoja voi ylittää - ilman että olisi mekanismi jossa uutta tietoa luotaisiin tyhjästä..

        Kaiken lisäksi jo olemassa olevien rakenteiden muuntelu ei kerro mitään niiden alkuperästä. Evoluutioteorialle olisi tärkeää vain tietää mekanismi biologisten rakenteiden alkuperälle mutta sitä ei nimenomaan tunneta ja tämän vuoksi koko teoria on vain suuri uskomusjärjestelmä.

        "Se että tieteellä ei ole selitystä biologian perusrakenteiden alkuperälle tarkoittaa sitä ettei tiede nimenomaan tunne mekanismia joka ne selittäisi."

        Hyvin monelle rakenteelle on löytynyt evolutiivinen selitys. Tiedätkö muuten, että ihmisen korvassa simpukassa oleva värekarvarakenne on hyvin samanlainen kalan kylkiviivan värekarvarakenteen kanssa?

        "geenivaraston muuntelu kapasiteetti on kuitenkin rajallinen eikä näitä rajoja voi ylittää "

        Kerropa se raja tai mekanismi, jolla se raja tulee vastaan? Kerropa se mekanismi, joka rajoittaa millään tavalla DNA-juosteen pituutta tai sisältöä.

        "Evoluutioteorialle olisi tärkeää vain tietää mekanismi biologisten rakenteiden alkuperälle..."

        Miksi se olisi tärkeää? Evoluutioteoria selittää vallan hyvin muuntelumekanismit sekä luonnonvalinnan ja siten selittää vallan hyvin sitten tätä nykyista havaittavaa biodiversiteettiä. Perimän muuntelua on havaittu, lajiutumisia on havaittu. Siten sekä ihmisen ja simpanssin perimästä löydetään yhtäläisyyksiä (ja eroavaisuuksia), joka selittää myös sen, miksi me olemme simpanssien kanssa kovin samannäköisiä ja -oloisia, mutta kuitenkin niin paljon simpansseista eroavia. Ja evoluutioteorian pohjalta löytyy myös se selitys sekä samalla tavalla rikkiolevasta C-vitamiinin tuottamiseen liittyvästä geenistä, että kaikille niistä yhteisistä ERV:stä kuten myös sille, miksi yksi ihmisen kromosomi on ilmiselvästi fuusioitunut kahdesta kromosomista (kun ne simpanssilla ovat erillisiä).

        "...mekanismi biologisten rakenteiden alkuperälle..."

        Jos ja kun ne biologiset rakenteet perustuvat perimän sisältöön, niin silloin se mekanismi on nimenomaan tuon perimän siirtymiseen liittyvät ilmiöt - kuten muuntelu ja luonnonvalinta. Kuten luonnosta havaitaan.


      • t-torppa kirjoitti:

        MUTAATIOT

        Kasvigeneetikko Dr John Sanford kertoo mutaatioista seuraavaa;

        - "Perimät ovat ne perinnölliset tekniset tiedot, jotka mahdollistavat elämän olemassaolon. Tekniset tiedot ovat selvästi luonnostaan tarkoin määrätty. Tämä tarkoittaa, että satunnaiset muutokset teknisissä tiedoissa sekoittavat tietoa erittäin suurella varmuudella."

        - "Mutaatiot ovat tekstinkäsittelyvirheitä solun käyttöohjeissa. Mutaatiot tuhoavat systemaattisesti geneettistä informaatiota – aivan kuten tekstinkäsittelyvirheet tuhoavat kirjoitettua informaatiota.. Mitä enemmän mutaatioita sitä vähemmän informaatiota. Tämä on mutaatioprosessin perusperiaate."

        Mutaatiot tuhoavat systemaattisesti geneettistä informaatiota - sillä ne ovat tekstinkäsittelyvirheitä solun käyttöohjeissa. Mutaatiot tuhoavat solun ohjeiden merkityksellisyyttä - koska satunnaiset muutokset teknisissä tiedoissa sekoittavat tietoa erittäin suurella varmuudella. Tässä valossa mutaatioita voisi verrata autoon kertyvään ruosteeseen. Autoon kertyy ajansaatossa ruostetta joka rappeuttaa - tuhoaa auton merkityksellisiä rakenteita. Mitä ruosteisempi auto sitä vähemmän on toiminnallista merkityksellisyyttä autossa. Mitä vanhempi auto sitä ruosteisempi. Ruosteella ei ole älyä eikä kykyä kirjoittaa auton rakennus ja toimintaohjeita samoin kun kirjoitusvirheet eivät kykene kirjoittamaan tietosanakirjaa.



        LUONNONVALINTA

        Dr John Sanford kertoo;

        - "Valinta auttaa kyllä. Se poistaa joukosta pahimmat mutaatiot. Tämä hidastaa mutaatioiden aiheuttamaa rappeutumista.

        Lisäksi, hyvin harvoin syntyy hyödyllinen mutaatio, jolla on tarpeeksi iso vaikutus tullakseen valituksi – josta seuraisi jokin sopeutunut muunnos tai jonkin asteista hienosäätöä. Tämä auttaa myös rappeutumisen hidastamisessa. Valinta kuitenkin karsii huonoista mutaatioista vain hyvin pienen murto-osan. Ylivoimainen enemmistö huonoista mutaatioista on sellaisia, joita kertyy jatkuvasti, ne ovat aivan liian hiuksenhienoja – niiden vaikutus on aivan liian pieni – jotta valinnalla olisi vaikutusta niihin. Kääntöpuolella taas lähes kaikki hyödylliset mutaatiot (siinä määrin, kuin niitä ilmaantuu) ovat riippumattomia valintaprosessista, koska ne poikkeuksetta aiheuttavat biologiseen toiminnallisuuteen vain pikkuriikkisiä lisäyksiä.

        Valinta hidastaa mutaatioista johtuvaa rappeutumista, mutta se ei itse asiassa kykene edes aloittamaan rappeutumisen pysäyttämistä. Joten jopa kiivaassa valinnassa, evoluutio kulkee väärään suuntaan – kohti sukupuuttoa!"

        Valinta pystyy vain hidastamaan mutaatioista johtuvaa rappeutumista - mutta ei kykene edes pysäyttämään sitä. Näin siksi että suurin osa mutaatioista ovat vain vähän haitallisia ja tämän vuoksi valinta ei näe niitä. Valinta näkee vain pahimmat mutaatiot ja tämän vuoksi vähän haitalliset mutaatiot ovat kuin ruoste autossa jota silmä ei aluksi näe mutta ajansaatossa niitä kertyy siinä määrin että ne aiheuttavat toiminnallisia häiriöitä.

        Kyseiset hyödylliset mutaatiot joita on vain murto-osa verrattuna haitallisiin ovat myös virheitä jossa informaatiota menetetään mutta näistä virheistä voi olla joissain oloissa hyötyä - siksi niitä kutsutaan hyödyllisiksi mutaatioiksi. Ne eivät kehitä uutta informaatiota vaan tuhoavat jo olemassa olevaa mutta joskus jo olemassa olevan informaation tuhoutuminen voi tuoda valintaedun.

        Esimerkiksi kun koppikselta tuhoutuu siipien rakennusohjeet mutaation seurauksena niin se ei lennä tuulisissa oloissa mereen ja huku. Näin siivet tuhonnut mutaatio voi olla hyödyllinen tietyissä (tuulisissa) oloissa mutta kyseessä oli tapahtuma jossa geneettistä merkityksellistä informaatiota menetettiin. Tällaisilla 'hyödyllisillä' mutaatioilla ei voida perustella mikrobista - ihmiseksi kehitystä sillä kyseinen kehitys tarvitsisi autokuormittain ennestään täysin uusia merkityksellisiä rakennus ja toimintaohjeita (geneettistä informaatiota).

        Auton kohdalla hyödyllinen mutaatio olosuhteisiin nähden olisi esimerkiksi sellainen tapahtuma jossa autosta otetaan takaspoileri pois jotta se kulkee suoralla lujempaa. Spoilerin poisto tuo tiettyihin olosuhteisiin etua mutta kyseessä oli jo olemassa olevan ominaisuuden hävittäminen ei uuden kehittäminen. Jo olemassa olevien rakenteiden hävittäminen on vastakohta sille mitä evoluutioteorian opettama kehitys tarvitsisi toimiakseensa.

        *Mutaatiot tuhoavat systemaattisesti geneettistä informaatiota - sillä ne ovat tekstinkäsittelyvirheitä solun käyttöohjeissa. Mutaatiot tuhoavat solun ohjeiden merkityksellisyyttä - koska satunnaiset muutokset teknisissä tiedoissa sekoittavat tietoa erittäin suurella varmuudella.*

        Valheistasi ja vääristelystä huolimatta luonnossa havaitaan jatkuvasti uusia hyödyllisiä mutaatioita, joiden esiintymistä populaatiossa luonnonvalinta edistää. Vähättelette ja ohjaatte huomiota ilmiön negatiivisiin puoliin, jotta saisitte mutaatioiden ja luonnonvalinnan hyödyllisen mekanismin salattua.

        Torppa, et jostakin syystä vastannut tähän kysymykseeni viimeksi:

        Oletko voinut aikuisenakin juoda maitoa?

        Et vastaa, koska olet valehteleva pelkuri.

        *Kääntöpuolella taas lähes kaikki hyödylliset mutaatiot (siinä määrin, kuin niitä ilmaantuu) ovat riippumattomia valintaprosessista, koska ne poikkeuksetta aiheuttavat biologiseen toiminnallisuuteen vain pikkuriikkisiä lisäyksiä.*

        Lisää valehtelua, vääristelyä ja vähättelyä. Jos mutaatio on todella hyödyllinen, se myös erottuu niin että luonnonvalinta suosii sitä - kuten esimerkisi Oahun wallabien maksan erittämä entsyymi (huom. UUSI ominaisuus / taito) joka mahdollisti myrkyllisten kasvien hyödyntämisen ravintona. Tai kyky juoda maitoa aikuisenakin (tuokin on UUSI ominaisuus), yms. yms.

        Et ole Torppa edelleenkään kertonut, mikä on se "rappeutunut" tai "tuhoutunut" ominaisuus, jonka ihminen menetti geeneistään siinä vaiheessa kun mutaatio mahdollisti maidonjuonnin aikuisenakin.

        Et voi myöntää ettei sellaista ominaisuutta ole olemassakaan, koska olet valehteleva pelkuri.

        *Jo olemassa olevien rakenteiden hävittäminen on vastakohta sille mitä evoluutioteorian opettama kehitys tarvitsisi toimiakseensa.*

        Todella paksua sontaa. Evoluutio ohjaa kehitystä sopivaksi vallitseviin olosuhteisiin, ei siihen suuntaan mihin tietämätön kreationisti väittää sen menevän.

        Jos jokin ominaisuus on turha vallitsevissa olosuhteissa, se häviää pikkuhiljaa. Sinullakin Torppa ihmisapinana on aika köykäinen karvapeite, mutta silti edelleen karvankohottajalihas jokaisen karvan juurella turkin pörhistämiseen.

        Vaikka kuinka yrität valehdella ja muuta väittää, ne karvankohottajalihakset ovat siellä ihossasi. Minkä fantasian te kreationistit saisitte kiskaistua sieltä pohjattomasta hatustanne tälle asialle? Mitä tarkoitusta varten Jumala loi ihmiselle karvankohottajalihakset?

        Et voi kenties vastata, koska juuri tähän asiaan sinulla ei ole spammata Leisolan teksteistä mitään.


      • "Tässä kokeellisen luonnontieteen expertti mikrobi ja entsyymiasiantuntija bioprosessitekniikan emeritusprofessori Matti Leisola kertoo haastattelussani ettei evoluutiolle mikrobeista ihmiseksi ole kokeellisten havaintojen mukaan mekanismia"

        Onhan sille, kuten sinulle on osoitettu. muuntelu ja luonnonvalinta. Esim. jääkaloille tuo prosessi tuotti uusia jäänestoproteiineja, kun ne sopeutuivat kylmiin vesiin, muistatko?


      • tieteenharrastaja
        salmiakkisaluki kirjoitti:

        Olisi edelleen kiinnostavaa tietää valehteleeko Leisola tahallaan vai vastoin parempaa tietoa.

        Hän vain käyttää kreationistista seebralogiikkaa sanoen mutkikkaalla tavalla, että ellei tiede tiedä kaikkea, niin se ei tiedä mitään. Minusta tuo on kyllä valehtelua.


      • t-torppa kirjoitti:

        Tämä 99 % on sinun itse keksimä uskomus - todellisuus on jotain täysin muuta.

        Bioprosessitekniikan emeritusprofessori Matti Leisola kirjoittaa kirjassaan evoluutiouskon ihmemaassa s,36 seuraavaa:

        - "Olen yli neljänkymmenen vuoden ajan käynyt elämän ja lajien syntyä koskevaa keskustelua tiedeyhteisön sisällä ja sen ulkopuolella. Lähes kaikki tuntemani sadat tutkijat myöntävät kahdenkeskisissä, luottamuksellisissa keskusteluissa, ettei tieteellä ole selitystä geneettisen kielen, proteiinien, solukalvojen, aineenvaihduntareaktioiden, säätelyjärjestelmien, solujen ja biologian perusrakenteiden alkuperälle."

        "- "Olen yli neljänkymmenen vuoden ajan käynyt elämän ja lajien syntyä koskevaa keskustelua tiedeyhteisön sisällä ja sen ulkopuolella. Lähes kaikki tuntemani sadat tutkijat myöntävät kahdenkeskisissä, luottamuksellisissa keskusteluissa, ettei tieteellä ole selitystä geneettisen kielen, proteiinien, solukalvojen, aineenvaihduntareaktioiden, säätelyjärjestelmien, solujen ja biologian perusrakenteiden alkuperälle."

        Eli he myöntävät että elämän syntyä ei vielä tunneta. Mutta he ovat varmasti kertoneet Leisolalle myös, että elämän kehitys yhteisestä kantamuodosta miljardien vuosien aikana on tieteellinen fakta. Leisola ei vain siitä mainitse mitään.


      • tieteenharrastaja
        Råger Mer kirjoitti:

        Tuhkimo-satu on täysin evoluutioteorian puolella. Siinäkin hiiret muuttuvat hevosiksi yliluonnollisen taian avulla.

        Tuohan on selvästi kertaluomisen pamautus, ei evoluutiota.


      • tieteenharrastaja
        t-torppa kirjoitti:

        MUTAATIOT

        Kasvigeneetikko Dr John Sanford kertoo mutaatioista seuraavaa;

        - "Perimät ovat ne perinnölliset tekniset tiedot, jotka mahdollistavat elämän olemassaolon. Tekniset tiedot ovat selvästi luonnostaan tarkoin määrätty. Tämä tarkoittaa, että satunnaiset muutokset teknisissä tiedoissa sekoittavat tietoa erittäin suurella varmuudella."

        - "Mutaatiot ovat tekstinkäsittelyvirheitä solun käyttöohjeissa. Mutaatiot tuhoavat systemaattisesti geneettistä informaatiota – aivan kuten tekstinkäsittelyvirheet tuhoavat kirjoitettua informaatiota.. Mitä enemmän mutaatioita sitä vähemmän informaatiota. Tämä on mutaatioprosessin perusperiaate."

        Mutaatiot tuhoavat systemaattisesti geneettistä informaatiota - sillä ne ovat tekstinkäsittelyvirheitä solun käyttöohjeissa. Mutaatiot tuhoavat solun ohjeiden merkityksellisyyttä - koska satunnaiset muutokset teknisissä tiedoissa sekoittavat tietoa erittäin suurella varmuudella. Tässä valossa mutaatioita voisi verrata autoon kertyvään ruosteeseen. Autoon kertyy ajansaatossa ruostetta joka rappeuttaa - tuhoaa auton merkityksellisiä rakenteita. Mitä ruosteisempi auto sitä vähemmän on toiminnallista merkityksellisyyttä autossa. Mitä vanhempi auto sitä ruosteisempi. Ruosteella ei ole älyä eikä kykyä kirjoittaa auton rakennus ja toimintaohjeita samoin kun kirjoitusvirheet eivät kykene kirjoittamaan tietosanakirjaa.



        LUONNONVALINTA

        Dr John Sanford kertoo;

        - "Valinta auttaa kyllä. Se poistaa joukosta pahimmat mutaatiot. Tämä hidastaa mutaatioiden aiheuttamaa rappeutumista.

        Lisäksi, hyvin harvoin syntyy hyödyllinen mutaatio, jolla on tarpeeksi iso vaikutus tullakseen valituksi – josta seuraisi jokin sopeutunut muunnos tai jonkin asteista hienosäätöä. Tämä auttaa myös rappeutumisen hidastamisessa. Valinta kuitenkin karsii huonoista mutaatioista vain hyvin pienen murto-osan. Ylivoimainen enemmistö huonoista mutaatioista on sellaisia, joita kertyy jatkuvasti, ne ovat aivan liian hiuksenhienoja – niiden vaikutus on aivan liian pieni – jotta valinnalla olisi vaikutusta niihin. Kääntöpuolella taas lähes kaikki hyödylliset mutaatiot (siinä määrin, kuin niitä ilmaantuu) ovat riippumattomia valintaprosessista, koska ne poikkeuksetta aiheuttavat biologiseen toiminnallisuuteen vain pikkuriikkisiä lisäyksiä.

        Valinta hidastaa mutaatioista johtuvaa rappeutumista, mutta se ei itse asiassa kykene edes aloittamaan rappeutumisen pysäyttämistä. Joten jopa kiivaassa valinnassa, evoluutio kulkee väärään suuntaan – kohti sukupuuttoa!"

        Valinta pystyy vain hidastamaan mutaatioista johtuvaa rappeutumista - mutta ei kykene edes pysäyttämään sitä. Näin siksi että suurin osa mutaatioista ovat vain vähän haitallisia ja tämän vuoksi valinta ei näe niitä. Valinta näkee vain pahimmat mutaatiot ja tämän vuoksi vähän haitalliset mutaatiot ovat kuin ruoste autossa jota silmä ei aluksi näe mutta ajansaatossa niitä kertyy siinä määrin että ne aiheuttavat toiminnallisia häiriöitä.

        Kyseiset hyödylliset mutaatiot joita on vain murto-osa verrattuna haitallisiin ovat myös virheitä jossa informaatiota menetetään mutta näistä virheistä voi olla joissain oloissa hyötyä - siksi niitä kutsutaan hyödyllisiksi mutaatioiksi. Ne eivät kehitä uutta informaatiota vaan tuhoavat jo olemassa olevaa mutta joskus jo olemassa olevan informaation tuhoutuminen voi tuoda valintaedun.

        Esimerkiksi kun koppikselta tuhoutuu siipien rakennusohjeet mutaation seurauksena niin se ei lennä tuulisissa oloissa mereen ja huku. Näin siivet tuhonnut mutaatio voi olla hyödyllinen tietyissä (tuulisissa) oloissa mutta kyseessä oli tapahtuma jossa geneettistä merkityksellistä informaatiota menetettiin. Tällaisilla 'hyödyllisillä' mutaatioilla ei voida perustella mikrobista - ihmiseksi kehitystä sillä kyseinen kehitys tarvitsisi autokuormittain ennestään täysin uusia merkityksellisiä rakennus ja toimintaohjeita (geneettistä informaatiota).

        Auton kohdalla hyödyllinen mutaatio olosuhteisiin nähden olisi esimerkiksi sellainen tapahtuma jossa autosta otetaan takaspoileri pois jotta se kulkee suoralla lujempaa. Spoilerin poisto tuo tiettyihin olosuhteisiin etua mutta kyseessä oli jo olemassa olevan ominaisuuden hävittäminen ei uuden kehittäminen. Jo olemassa olevien rakenteiden hävittäminen on vastakohta sille mitä evoluutioteorian opettama kehitys tarvitsisi toimiakseensa.

        Kyllä Sanford olisi sinua auttanut jo ajat sitten, jos kykenisi. Turha enää toistella.


      • t-torppa kirjoitti:

        MUTAATIOT

        Kasvigeneetikko Dr John Sanford kertoo mutaatioista seuraavaa;

        - "Perimät ovat ne perinnölliset tekniset tiedot, jotka mahdollistavat elämän olemassaolon. Tekniset tiedot ovat selvästi luonnostaan tarkoin määrätty. Tämä tarkoittaa, että satunnaiset muutokset teknisissä tiedoissa sekoittavat tietoa erittäin suurella varmuudella."

        - "Mutaatiot ovat tekstinkäsittelyvirheitä solun käyttöohjeissa. Mutaatiot tuhoavat systemaattisesti geneettistä informaatiota – aivan kuten tekstinkäsittelyvirheet tuhoavat kirjoitettua informaatiota.. Mitä enemmän mutaatioita sitä vähemmän informaatiota. Tämä on mutaatioprosessin perusperiaate."

        Mutaatiot tuhoavat systemaattisesti geneettistä informaatiota - sillä ne ovat tekstinkäsittelyvirheitä solun käyttöohjeissa. Mutaatiot tuhoavat solun ohjeiden merkityksellisyyttä - koska satunnaiset muutokset teknisissä tiedoissa sekoittavat tietoa erittäin suurella varmuudella. Tässä valossa mutaatioita voisi verrata autoon kertyvään ruosteeseen. Autoon kertyy ajansaatossa ruostetta joka rappeuttaa - tuhoaa auton merkityksellisiä rakenteita. Mitä ruosteisempi auto sitä vähemmän on toiminnallista merkityksellisyyttä autossa. Mitä vanhempi auto sitä ruosteisempi. Ruosteella ei ole älyä eikä kykyä kirjoittaa auton rakennus ja toimintaohjeita samoin kun kirjoitusvirheet eivät kykene kirjoittamaan tietosanakirjaa.



        LUONNONVALINTA

        Dr John Sanford kertoo;

        - "Valinta auttaa kyllä. Se poistaa joukosta pahimmat mutaatiot. Tämä hidastaa mutaatioiden aiheuttamaa rappeutumista.

        Lisäksi, hyvin harvoin syntyy hyödyllinen mutaatio, jolla on tarpeeksi iso vaikutus tullakseen valituksi – josta seuraisi jokin sopeutunut muunnos tai jonkin asteista hienosäätöä. Tämä auttaa myös rappeutumisen hidastamisessa. Valinta kuitenkin karsii huonoista mutaatioista vain hyvin pienen murto-osan. Ylivoimainen enemmistö huonoista mutaatioista on sellaisia, joita kertyy jatkuvasti, ne ovat aivan liian hiuksenhienoja – niiden vaikutus on aivan liian pieni – jotta valinnalla olisi vaikutusta niihin. Kääntöpuolella taas lähes kaikki hyödylliset mutaatiot (siinä määrin, kuin niitä ilmaantuu) ovat riippumattomia valintaprosessista, koska ne poikkeuksetta aiheuttavat biologiseen toiminnallisuuteen vain pikkuriikkisiä lisäyksiä.

        Valinta hidastaa mutaatioista johtuvaa rappeutumista, mutta se ei itse asiassa kykene edes aloittamaan rappeutumisen pysäyttämistä. Joten jopa kiivaassa valinnassa, evoluutio kulkee väärään suuntaan – kohti sukupuuttoa!"

        Valinta pystyy vain hidastamaan mutaatioista johtuvaa rappeutumista - mutta ei kykene edes pysäyttämään sitä. Näin siksi että suurin osa mutaatioista ovat vain vähän haitallisia ja tämän vuoksi valinta ei näe niitä. Valinta näkee vain pahimmat mutaatiot ja tämän vuoksi vähän haitalliset mutaatiot ovat kuin ruoste autossa jota silmä ei aluksi näe mutta ajansaatossa niitä kertyy siinä määrin että ne aiheuttavat toiminnallisia häiriöitä.

        Kyseiset hyödylliset mutaatiot joita on vain murto-osa verrattuna haitallisiin ovat myös virheitä jossa informaatiota menetetään mutta näistä virheistä voi olla joissain oloissa hyötyä - siksi niitä kutsutaan hyödyllisiksi mutaatioiksi. Ne eivät kehitä uutta informaatiota vaan tuhoavat jo olemassa olevaa mutta joskus jo olemassa olevan informaation tuhoutuminen voi tuoda valintaedun.

        Esimerkiksi kun koppikselta tuhoutuu siipien rakennusohjeet mutaation seurauksena niin se ei lennä tuulisissa oloissa mereen ja huku. Näin siivet tuhonnut mutaatio voi olla hyödyllinen tietyissä (tuulisissa) oloissa mutta kyseessä oli tapahtuma jossa geneettistä merkityksellistä informaatiota menetettiin. Tällaisilla 'hyödyllisillä' mutaatioilla ei voida perustella mikrobista - ihmiseksi kehitystä sillä kyseinen kehitys tarvitsisi autokuormittain ennestään täysin uusia merkityksellisiä rakennus ja toimintaohjeita (geneettistä informaatiota).

        Auton kohdalla hyödyllinen mutaatio olosuhteisiin nähden olisi esimerkiksi sellainen tapahtuma jossa autosta otetaan takaspoileri pois jotta se kulkee suoralla lujempaa. Spoilerin poisto tuo tiettyihin olosuhteisiin etua mutta kyseessä oli jo olemassa olevan ominaisuuden hävittäminen ei uuden kehittäminen. Jo olemassa olevien rakenteiden hävittäminen on vastakohta sille mitä evoluutioteorian opettama kehitys tarvitsisi toimiakseensa.

        Sanfordin väittämät kumoutuvat yksinkertaisella esimerkillä: jääkaloilla.


      • tieteenharrastaja
        moloch_horridus kirjoitti:

        "Tässä kokeellisen luonnontieteen expertti mikrobi ja entsyymiasiantuntija bioprosessitekniikan emeritusprofessori Matti Leisola kertoo haastattelussani ettei evoluutiolle mikrobeista ihmiseksi ole kokeellisten havaintojen mukaan mekanismia"

        Onhan sille, kuten sinulle on osoitettu. muuntelu ja luonnonvalinta. Esim. jääkaloille tuo prosessi tuotti uusia jäänestoproteiineja, kun ne sopeutuivat kylmiin vesiin, muistatko?

        Tarkoittaakohan hän, ettei ole mekanismia sille, että mikrobista kehittyy nimenomaan ja ennalta tavoitellusti ihminen eikä vain jotakin, mikä sattui tulemaan. Silloin hän taitaisi olla oikeassa, mutta evoluutioteoriahan ei tuollaista väitä.


      • Råger Mer
        moloch_horridus kirjoitti:

        Sanfordin väittämät kumoutuvat yksinkertaisella esimerkillä: jääkaloilla.

        Tuliko niistä jääkaloista uusi laji vai pysyikö ne edelleen kaloina ja oliko ne kaloja ennen kuin niistä tuli jääkaloja?


      • tieteenhaastaja
        tieteenharrastaja kirjoitti:

        Tarkoittaakohan hän, ettei ole mekanismia sille, että mikrobista kehittyy nimenomaan ja ennalta tavoitellusti ihminen eikä vain jotakin, mikä sattui tulemaan. Silloin hän taitaisi olla oikeassa, mutta evoluutioteoriahan ei tuollaista väitä.

        Miksi evokit tekevät liian yksinkertaistettuja päätelmiä asioista jotka ovat äärettömän monimutkaisia?
        Ja tämän palstan evokit ovat siinä kaikkein heikoimmin välinein liikkeellä.
        Yksi erikoisuus evokkiudessa on myös se, että kummallisesti jokainen kristilliseen uskoon kääntynyt tiedemies hylkää evokkiuden, mutta ei oikeiden tieteiden päätelmiä.
        Lapsesta asti kirkkoihin kuuluneista perinnäistapauskovista en nyt puhu mitään.


      • tiedosta kiitolline
        antimytomaani_orig kirjoitti:

        "Se että tieteellä ei ole selitystä biologian perusrakenteiden alkuperälle tarkoittaa sitä ettei tiede nimenomaan tunne mekanismia joka ne selittäisi."

        Hyvin monelle rakenteelle on löytynyt evolutiivinen selitys. Tiedätkö muuten, että ihmisen korvassa simpukassa oleva värekarvarakenne on hyvin samanlainen kalan kylkiviivan värekarvarakenteen kanssa?

        "geenivaraston muuntelu kapasiteetti on kuitenkin rajallinen eikä näitä rajoja voi ylittää "

        Kerropa se raja tai mekanismi, jolla se raja tulee vastaan? Kerropa se mekanismi, joka rajoittaa millään tavalla DNA-juosteen pituutta tai sisältöä.

        "Evoluutioteorialle olisi tärkeää vain tietää mekanismi biologisten rakenteiden alkuperälle..."

        Miksi se olisi tärkeää? Evoluutioteoria selittää vallan hyvin muuntelumekanismit sekä luonnonvalinnan ja siten selittää vallan hyvin sitten tätä nykyista havaittavaa biodiversiteettiä. Perimän muuntelua on havaittu, lajiutumisia on havaittu. Siten sekä ihmisen ja simpanssin perimästä löydetään yhtäläisyyksiä (ja eroavaisuuksia), joka selittää myös sen, miksi me olemme simpanssien kanssa kovin samannäköisiä ja -oloisia, mutta kuitenkin niin paljon simpansseista eroavia. Ja evoluutioteorian pohjalta löytyy myös se selitys sekä samalla tavalla rikkiolevasta C-vitamiinin tuottamiseen liittyvästä geenistä, että kaikille niistä yhteisistä ERV:stä kuten myös sille, miksi yksi ihmisen kromosomi on ilmiselvästi fuusioitunut kahdesta kromosomista (kun ne simpanssilla ovat erillisiä).

        "...mekanismi biologisten rakenteiden alkuperälle..."

        Jos ja kun ne biologiset rakenteet perustuvat perimän sisältöön, niin silloin se mekanismi on nimenomaan tuon perimän siirtymiseen liittyvät ilmiöt - kuten muuntelu ja luonnonvalinta. Kuten luonnosta havaitaan.

        Perimäksi ei voi sanoa sellaista joita ei ollut aiemmin kenelläkään!
        Vai mitä muuta voi periä niin ettei sitä ollut olemassakaan ennemmin?
        Miksi evoluutioteoria poikkeaa monin tavoin kaikista muista asioista sisältäen poikkeuksia joita sitten selitetään tulkinnoin eikä muiden tieteiden tuntemilla lainalaisuuksilla?


      • Råger Mer kirjoitti:

        Tuliko niistä jääkaloista uusi laji vai pysyikö ne edelleen kaloina ja oliko ne kaloja ennen kuin niistä tuli jääkaloja?

        "Tuliko niistä jääkaloista uusi laji vai pysyikö ne edelleen kaloina ja oliko ne kaloja ennen kuin niistä tuli jääkaloja?"

        Niistä tuli uusi heimo, johon kuuluu ainakin n. 25 tähän mennessä tunnistettua lajia.


      • tieteenharrastaja kirjoitti:

        Tarkoittaakohan hän, ettei ole mekanismia sille, että mikrobista kehittyy nimenomaan ja ennalta tavoitellusti ihminen eikä vain jotakin, mikä sattui tulemaan. Silloin hän taitaisi olla oikeassa, mutta evoluutioteoriahan ei tuollaista väitä.

        "Tarkoittaakohan hän, ettei ole mekanismia sille, että mikrobista kehittyy nimenomaan ja ennalta tavoitellusti ihminen eikä vain jotakin, mikä sattui tulemaan. Silloin hän taitaisi olla oikeassa, mutta evoluutioteoriahan ei tuollaista väitä."

        Niinpä. Mutta veikkaan kuitenkin, että hän on valikoinut sanansa niin, että yksinkertaisimmat kuulijansa luulevat hänen tarkoitavan, ettei evoluutiolla olisi mekanismia.


      • tiedosta kiitolline kirjoitti:

        Perimäksi ei voi sanoa sellaista joita ei ollut aiemmin kenelläkään!
        Vai mitä muuta voi periä niin ettei sitä ollut olemassakaan ennemmin?
        Miksi evoluutioteoria poikkeaa monin tavoin kaikista muista asioista sisältäen poikkeuksia joita sitten selitetään tulkinnoin eikä muiden tieteiden tuntemilla lainalaisuuksilla?

        *Perimäksi ei voi sanoa sellaista joita ei ollut aiemmin kenelläkään!*

        Ei kai siellä ole uskovainen taas latelemassa sääntöjä ja rajoituksia tieteellisille termeille?

        Tiedoksesi: perimä on se geneettinen potti minkä jälkeläinen vanhemmaltaan tai vanhemmiltaan saa, riippumatta sen ominaisuuksista.

        *Vai mitä muuta voi periä niin ettei sitä ollut olemassakaan ennemmin?*

        Väännetään rautalangasta: perit (kuvitteellisesti) isältäsi uistinpakin, jossa on paljon vaappuja. Perintötavaroita järjestelevä muuttofirma käsittelee uistinpakkia huolimattomasti sillä seurauksella että erään vaapun uintiläppä vääntyy hieman. Vaapusta tulee tuon
        "vahingon" seurauksena kuitenkin erittäin hyväuintinen, pakin parhaiten saalista antava viehe.

        Olet siis perinyt isäsi uistimen, jossa on sattuman seuraksena hieman parempi ominaisuus kuin siinä oli koskaan isäsi sitä käyttäessä. Sama voi käydä geenien ohjaamille ominaisuuksille.

        *Miksi evoluutioteoria poikkeaa monin tavoin kaikista muista asioista sisältäen poikkeuksia joita sitten selitetään tulkinnoin eikä muiden tieteiden tuntemilla lainalaisuuksilla?*

        Ja nämä poikkeukset ovat tarkalleen ottaen mitä?


      • Råger Mer kirjoitti:

        Tuliko niistä jääkaloista uusi laji vai pysyikö ne edelleen kaloina ja oliko ne kaloja ennen kuin niistä tuli jääkaloja?

        *Tuliko niistä jääkaloista uusi laji vai pysyikö ne edelleen kaloina ja oliko ne kaloja ennen kuin niistä tuli jääkaloja?*

        Oliko nyt niin, että sinun tulkintasi mukaan maapallolla on olemassa yksi kalalaji, jonka lajinimi on...niinkin valtavan eksoottinen ja tieteellinen kuin "kala"?

        Ymmärrän koko ajan paremmin, miksi monet biologit eivät keskustele tai väittele kreationistien kanssa lainkaan.


      • bg-ope kirjoitti:

        *Tuliko niistä jääkaloista uusi laji vai pysyikö ne edelleen kaloina ja oliko ne kaloja ennen kuin niistä tuli jääkaloja?*

        Oliko nyt niin, että sinun tulkintasi mukaan maapallolla on olemassa yksi kalalaji, jonka lajinimi on...niinkin valtavan eksoottinen ja tieteellinen kuin "kala"?

        Ymmärrän koko ajan paremmin, miksi monet biologit eivät keskustele tai väittele kreationistien kanssa lainkaan.

        >Ymmärrän koko ajan paremmin, miksi monet biologit eivät keskustele tai väittele kreationistien kanssa lainkaan.

        Taitaa vain hyvin harva lähteä moiseen ajanhaaskuuseen. Mukava kuitenkin että olet poikkeus.


      • tieteenhaastaja kirjoitti:

        Miksi evokit tekevät liian yksinkertaistettuja päätelmiä asioista jotka ovat äärettömän monimutkaisia?
        Ja tämän palstan evokit ovat siinä kaikkein heikoimmin välinein liikkeellä.
        Yksi erikoisuus evokkiudessa on myös se, että kummallisesti jokainen kristilliseen uskoon kääntynyt tiedemies hylkää evokkiuden, mutta ei oikeiden tieteiden päätelmiä.
        Lapsesta asti kirkkoihin kuuluneista perinnäistapauskovista en nyt puhu mitään.

        >Yksi erikoisuus evokkiudessa on myös se, että kummallisesti jokainen kristilliseen uskoon kääntynyt tiedemies hylkää evokkiuden, mutta ei oikeiden tieteiden päätelmiä.

        Ja biologeissa sekä muissa luonnontieteilijöissä on vaikka kuinka paljon kristittyjä, joiden ei tarvitse koskaan kääntyä mihinkään.

        Sinä Perähikiä puhut tietysti lahkofundamentalismiin kääntyneistä, ja heitä on tiedemiehissä erittäin vähän, joskin niitä vähiä näköjään eräät tollot lainaavat maanisen kiihkeästi.


      • moloch_horridus kirjoitti:

        "Tarkoittaakohan hän, ettei ole mekanismia sille, että mikrobista kehittyy nimenomaan ja ennalta tavoitellusti ihminen eikä vain jotakin, mikä sattui tulemaan. Silloin hän taitaisi olla oikeassa, mutta evoluutioteoriahan ei tuollaista väitä."

        Niinpä. Mutta veikkaan kuitenkin, että hän on valikoinut sanansa niin, että yksinkertaisimmat kuulijansa luulevat hänen tarkoitavan, ettei evoluutiolla olisi mekanismia.

        Evoluutiolla ei ole mekanismia ja tämä on sinulle jo monesti selitetty. Se että kilpa-auton säätöjä voidaan muunnella tietyissä variaatio rajoissa ei kerro mitään prosessista joka auton rakensi.

        Kaikki evolutionistien todisteet ovat vain tällaisia jo olemassa olevien rakenteiden muuntelua tietyissä variaatio rajoissa - teidän todisteet eivät kuitenkaan todista mitään miten rakenteet alunperin ovat saaneet alkunsa.

        Puhutte jo olemassa olevien rakenteiden ja ohjeiden säätelystä kun keskustelua käydään rakenteiden alkuperästä. Joudutte tekemään näin siksi ettei evoluutiolla ole mekanismia kehittää uusia rakenteita ja siksi mikrobista ei ole voinut kehittyä mikään muu kuin mikrobi.


      • Råger Mer
        moloch_horridus kirjoitti:

        "Tuliko niistä jääkaloista uusi laji vai pysyikö ne edelleen kaloina ja oliko ne kaloja ennen kuin niistä tuli jääkaloja?"

        Niistä tuli uusi heimo, johon kuuluu ainakin n. 25 tähän mennessä tunnistettua lajia.

        Eli ne pysyi kaloina eikä mitään evoluutiota tapahtunut.

        Heimo Holopainen on muusikko mutta ei ole eri lajia kuin muut ihmiset.


      • Totuus esiin
        t-torppa kirjoitti:

        Evoluutiolla ei ole mekanismia ja tämä on sinulle jo monesti selitetty. Se että kilpa-auton säätöjä voidaan muunnella tietyissä variaatio rajoissa ei kerro mitään prosessista joka auton rakensi.

        Kaikki evolutionistien todisteet ovat vain tällaisia jo olemassa olevien rakenteiden muuntelua tietyissä variaatio rajoissa - teidän todisteet eivät kuitenkaan todista mitään miten rakenteet alunperin ovat saaneet alkunsa.

        Puhutte jo olemassa olevien rakenteiden ja ohjeiden säätelystä kun keskustelua käydään rakenteiden alkuperästä. Joudutte tekemään näin siksi ettei evoluutiolla ole mekanismia kehittää uusia rakenteita ja siksi mikrobista ei ole voinut kehittyä mikään muu kuin mikrobi.

        Evoluutioteoria ei käsittele sitä, miten ensimmäinen solu ja siinä olevat mekanismit toimivat.

        Tiedän kyllä, että kretiineillä on tapa määritellä kaikki asiat uudelleen, etämä on heidän tapansa vääristellä ja valehdella.


        Kerro esimerkiksi, mitä uusia rakenteita on tarvittu, kun ihminen ja simpanssi ovat evolutiivisesti eronneet eri haaroihin?
        Et kai suinkaan väitä, että olemme samaa lajia?


      • Totuus esiin kirjoitti:

        Evoluutioteoria ei käsittele sitä, miten ensimmäinen solu ja siinä olevat mekanismit toimivat.

        Tiedän kyllä, että kretiineillä on tapa määritellä kaikki asiat uudelleen, etämä on heidän tapansa vääristellä ja valehdella.


        Kerro esimerkiksi, mitä uusia rakenteita on tarvittu, kun ihminen ja simpanssi ovat evolutiivisesti eronneet eri haaroihin?
        Et kai suinkaan väitä, että olemme samaa lajia?

        Evoluutioon uskovat opettavat että pelikaanit, granaattiomenat, ihmiset ja ponit ovat saaneet alkunsa yksisoluisista eliöistä (yksisoluisia eliöitä ovat esimerkiksi alkueläimet). Jokaisessa tapauksessa DNA- ”reseptin” on täytynyt läpikäydä valtava informaation lisääntyminen väitettyjen miljoonien vuosien aikana. Yksisoluisella eliöllä ei ole ohjeita silmien, korvien, veren, ihon, kavioiden, aivojen jne. valmistamiseen joita ponit tarvitsevat. Jotta ponit olisivat voineet saada alkunsa alkueläimistä täytyisi olla jokin mekanismi joka synnyttää uutta informaatiota.

        Evoluutioon uskovat tervehtivät luonnonvalintaa kuin se olisi luova jumalatar mutta tosiasia on että valinta sellaisenaan aina hankkiutuu eroon informaatiosta ei koskaan päinvastoin. Jotta olisi jokin tapa informaation lisäämiseksi evoluutioon tosiuskovien ainoa vaihtoehto ovat geneettiset kopiointivirheet tai vahingot eli sattumanvaraiset mutaatiot (joita valinta voi sitten ”suodattaa”). Ongelma on kuitenkin siinä että jos mutaatiot kykenisivät lisäämään tarvittavaa informaatiota meidän tulisi nähdä ympärillämme satoja esimerkkejä – ottaen huomioon, että jatkuvasti tapahtuu useita tuhansia mutaatioita. Mutta aina kun tutkimme mutaatioita, osoittautuu poikkeuksetta, että kyse on informaation menettämisestä tai huonontumisesta. Tämä on totta jopa niissä harvoissa tapauksissa joissa mutaation aiheuttama vika saa aikaan eloonjäämisedun – esimerkiksi kun tuulisilla saarilla elävät kovakuoriaiset menettävät siipensä. (katso lentokyvyttömät kovakuoriaiset - http://www.luominen.fi/genetiikka/lentokyvyttomat-kovakuoriaiset ).

        Eliöiden erilaistuessa geenistö hupenee lisääntyvässä määrin. Mitä enemmän eliöt sopeutuvat valinnan kautta ympäristöihinsä, eli mitä erikoistuneemmiksi ne tulevat, sitä pienempi osa niillä on jäljellä niiden luodulle lajille alun perin luodusta informaatiovarastosta. Siten luonnonvalinnan käytettävissä on vähemmän informaatiota tulevaisuudessa jotta se voisi uudelleen sopeutua jos olosuhteet muuttuisivat. Vähemmän mukautuvat, vähemmän sopeutumaan kykenevät populaatiot ovat selvästi kulkemassa kohti sukupuuttoa eivät kehittymässä.



        Muista että evoluutiouskomukset opettavat että olipa kerran eläviä olentoja mutta ei keuhkoja – keuhkot eivät olleet vielä kehittyneet joten ei ollut DNA-informaatiota joka olisi koodannut keuhkojen valmistamista. Jotenkin tämä ohjelma oli kirjoitettava. Oli synnyttävä uutta informaatiota jota ei aikaisemmin ollut olemassa - ei missään.

        Myöhemmin oli olemassa keuhkoja mutta missään maailmassa ei ollut höyheniä eikä siten geneettistä informaatiota höyhenille. Todellisen maailman tarkkailu on musertavalla tavalla osoittanut mutaation olevan täysin kykenemätön syöttämään järjestelmään tarvitun uuden informaation. Itse asiassa mutaatiot kiihdyttävät kokonaisuutena alaspäin menevää suuntausta lisäämällä geneettistä taakkaa vahingollisten mutaatioiden muodossa joita meihin kaikkiin on sukupolvien aikana kertynyt satamäärin.

        Toisin sanoen populaatiot voivat muuttua ja sopeutua koska niillä on paljon informaatiota (muuntelu kykyä) DNA- ”reseptissään”. Mutta elleivät mutaatiot kykene syöttämään perimään uutta informaatiota joka kerta kun muuntelua tai sopeutumista tapahtuu kokonaisinformaatio vähenee (kun valinta karsii sopeutumattomat osat populaatiosta osa informaatiosta menetetään tuossa populaatiossa).

        Siten kun informaatiota on tietty määrä niin mitä enemmän näemme sopeutumista sitä pienempi mahdollisuus sopeutumiseen on tulevaisuudessa. Evoluutiojuna suuntaa kulkunsa varmuudella alaspäin ja se on matkalla sukupuuton laituriin!


      • t-torppa kirjoitti:

        Evoluutiolla ei ole mekanismia ja tämä on sinulle jo monesti selitetty. Se että kilpa-auton säätöjä voidaan muunnella tietyissä variaatio rajoissa ei kerro mitään prosessista joka auton rakensi.

        Kaikki evolutionistien todisteet ovat vain tällaisia jo olemassa olevien rakenteiden muuntelua tietyissä variaatio rajoissa - teidän todisteet eivät kuitenkaan todista mitään miten rakenteet alunperin ovat saaneet alkunsa.

        Puhutte jo olemassa olevien rakenteiden ja ohjeiden säätelystä kun keskustelua käydään rakenteiden alkuperästä. Joudutte tekemään näin siksi ettei evoluutiolla ole mekanismia kehittää uusia rakenteita ja siksi mikrobista ei ole voinut kehittyä mikään muu kuin mikrobi.

        >Evoluutiolla ei ole mekanismia ja tämä on sinulle jo monesti selitetty.

        Selitäpä se vielä biologeillekin. Ovat varmaan haltioissaan. Muista liittää ansioluettelosi ja Leisolan vahvistus mukaan.

        Joko muuten olet löytänyt Israelin kartalta?


      • muItinikki
        RepeRuutikallo kirjoitti:

        >Evoluutiolla ei ole mekanismia ja tämä on sinulle jo monesti selitetty.

        Selitäpä se vielä biologeillekin. Ovat varmaan haltioissaan. Muista liittää ansioluettelosi ja Leisolan vahvistus mukaan.

        Joko muuten olet löytänyt Israelin kartalta?

        Te ateisti hihhulit olette pilkannut Torppaa täällä joka päivä mutta Torppa on todistanut perusteellisesti miten valheellinen evoluutioteoria on.

        Evoluutioteoria tulee aina jäämään teoria asteelle ja se tullaan aina muistamaan Tuhkimo-sadun uudenaikaisena versiona.


      • bg-ope kirjoitti:

        *Perimäksi ei voi sanoa sellaista joita ei ollut aiemmin kenelläkään!*

        Ei kai siellä ole uskovainen taas latelemassa sääntöjä ja rajoituksia tieteellisille termeille?

        Tiedoksesi: perimä on se geneettinen potti minkä jälkeläinen vanhemmaltaan tai vanhemmiltaan saa, riippumatta sen ominaisuuksista.

        *Vai mitä muuta voi periä niin ettei sitä ollut olemassakaan ennemmin?*

        Väännetään rautalangasta: perit (kuvitteellisesti) isältäsi uistinpakin, jossa on paljon vaappuja. Perintötavaroita järjestelevä muuttofirma käsittelee uistinpakkia huolimattomasti sillä seurauksella että erään vaapun uintiläppä vääntyy hieman. Vaapusta tulee tuon
        "vahingon" seurauksena kuitenkin erittäin hyväuintinen, pakin parhaiten saalista antava viehe.

        Olet siis perinyt isäsi uistimen, jossa on sattuman seuraksena hieman parempi ominaisuus kuin siinä oli koskaan isäsi sitä käyttäessä. Sama voi käydä geenien ohjaamille ominaisuuksille.

        *Miksi evoluutioteoria poikkeaa monin tavoin kaikista muista asioista sisältäen poikkeuksia joita sitten selitetään tulkinnoin eikä muiden tieteiden tuntemilla lainalaisuuksilla?*

        Ja nämä poikkeukset ovat tarkalleen ottaen mitä?

        Mitä uusia rakenteita uistinpakkiin syntyi? Jo olemassa oleva rakenne vain vääntyi - eli mitään uutta rakennetta ei syntynyt.

        Se että vaappu ui paremmin ja siksi saa etua muihin vaappuhin nähden ei tee vaappuun uusia monimutkaisia rakenteita jota evoluutio tarvitsisi jotta mikrobeista ihmiseksi kehitys olisi mahdollista.


      • muItinikki
        Totuus esiin kirjoitti:

        Evoluutioteoria ei käsittele sitä, miten ensimmäinen solu ja siinä olevat mekanismit toimivat.

        Tiedän kyllä, että kretiineillä on tapa määritellä kaikki asiat uudelleen, etämä on heidän tapansa vääristellä ja valehdella.


        Kerro esimerkiksi, mitä uusia rakenteita on tarvittu, kun ihminen ja simpanssi ovat evolutiivisesti eronneet eri haaroihin?
        Et kai suinkaan väitä, että olemme samaa lajia?

        (mitä uusia rakenteita on tarvittu, kun ihminen ja simpanssi ovat evolutiivisesti eronneet eri haaroihin?)

        Ihminen ja apina ei ole eronneet eri haaroihin vaan apina on luotu apinaksi ja ihminen ihmiseksi.
        Jos haluatte lukea evoluutiosta niin lukekaa Tuhkimo-satu, siinä kerrotaan miten lajit muuttuvat toiseksi lajiksi ateistien mukaan.


      • muItinikki kirjoitti:

        Te ateisti hihhulit olette pilkannut Torppaa täällä joka päivä mutta Torppa on todistanut perusteellisesti miten valheellinen evoluutioteoria on.

        Evoluutioteoria tulee aina jäämään teoria asteelle ja se tullaan aina muistamaan Tuhkimo-sadun uudenaikaisena versiona.

        >Te ateisti hihhulit olette pilkannut Torppaa täällä joka päivä

        Hänen omasta pyynnöstään. Ei kai kukaan ole häntä pakottanut tänne esittelemään tyhmyyttään ja pilkattavaksi? Jos on, pakottaja hävetköön moista ja tulkoon itse tekemään itsestään naurunalaisen. Arvostelukyvyttömien hölmöjen hyväksikäyttäminen on moraaalitonta touhua.

        >Torppa on todistanut perusteellisesti miten valheellinen evoluutioteoria on.

        Torppa on todistanut perusteellisesti, ettei tiedä mistään mitään. Juuri nyt hän varmaan etsii Israelia kartalta. Mene ihmeessä auttamaan uskonveljeäsi, sillä voi olla että hän ei selviä niin vaativasta urakasta yksin.

        >Evoluutioteoria tulee aina jäämään teoria asteelle

        Niin tietysti, kuinkas muutenkaan? Onhan "teoria aste" korkein tieteellisen selityksen taso.


      • Råger Mer kirjoitti:

        Eli ne pysyi kaloina eikä mitään evoluutiota tapahtunut.

        Heimo Holopainen on muusikko mutta ei ole eri lajia kuin muut ihmiset.

        "Eli ne pysyi kaloina eikä mitään evoluutiota tapahtunut."

        Haha. Pelleilet. Yrität esittää kreationistit mahdollisimman typerinä. Aivan kuin maailmassa muka oli vain yksi kalalaji. Eivvät edes kreationistit tietenkään noin hölmöjä voi olla.

        "Heimo Holopainen on muusikko mutta ei ole eri lajia kuin muut ihmiset."

        Niin, nykyisin on elossa vain yksi ihmislaji. Esim. 50 000 vuotta sitten oli elossa ainakin kolme, mahdollisesti viisi eri ihmislajia.


      • t-torppa kirjoitti:

        Evoluutiolla ei ole mekanismia ja tämä on sinulle jo monesti selitetty. Se että kilpa-auton säätöjä voidaan muunnella tietyissä variaatio rajoissa ei kerro mitään prosessista joka auton rakensi.

        Kaikki evolutionistien todisteet ovat vain tällaisia jo olemassa olevien rakenteiden muuntelua tietyissä variaatio rajoissa - teidän todisteet eivät kuitenkaan todista mitään miten rakenteet alunperin ovat saaneet alkunsa.

        Puhutte jo olemassa olevien rakenteiden ja ohjeiden säätelystä kun keskustelua käydään rakenteiden alkuperästä. Joudutte tekemään näin siksi ettei evoluutiolla ole mekanismia kehittää uusia rakenteita ja siksi mikrobista ei ole voinut kehittyä mikään muu kuin mikrobi.

        "Evoluutiolla ei ole mekanismia ja tämä on sinulle jo monesti selitetty."

        Evoluutiolla on mekanismit, kuten sinulle on jo monta kertaa selitetty ja osoitettu esimerkein. Sinä itse vain kieltäydyt ymmärtämästä niitä.

        "Se että kilpa-auton säätöjä voidaan muunnella tietyissä variaatio rajoissa ei kerro mitään prosessista joka auton rakensi."

        Vertauksesi koskee lähinnä elämän alkua. Nyt kuitenkin on kysymys evoluutiosta, ei elämän alusta.

        "Kaikki evolutionistien todisteet ovat vain tällaisia jo olemassa olevien rakenteiden muuntelua tietyissä variaatio rajoissa - teidän todisteet eivät kuitenkaan todista mitään miten rakenteet alunperin ovat saaneet alkunsa."

        Evoluutio nyt vain sattuu olemaan olemassa olevien rakenteiden muuntelua. Ja joskus tuo muuntelu tuottaa uusia rakenteita. Näytin sinulle jo tieteellisen tutkimuksen, joka todisti jääkalojen jäänestoproteiinien alkuperäksi erään haimaentsyymiä tuottavan geenin. Sinä itse vain kieltäydyit hyväksymästä tuota tutkimusta selittämällä, että se olisi menneisyyttä koskeva uskomus, vaikka sinulla ei ollut mitään perustetta esittää tutkimusta vääräksi. Vasta jos kykenet osoittamaan, että missä kohtaa tutkimuksessa on virhe, niin voit väittää, että jäänestoproteiinien evoluutio on mahdotonta ja että esimerkit, joita sinulle on annettu eivät tue sitä, että evoluutiolla olisi mekanismi. Nyt olet sen saanut eikä sinulla ole perustetta sitä kieltää.

        "Puhutte jo olemassa olevien rakenteiden ja ohjeiden säätelystä kun keskustelua käydään rakenteiden alkuperästä."

        Hohhoh. Et sinä keskustele, sinä paasaat ja saarnaat. Rehellinen keskustelu olisi sellaista, jossa yhdessä etsitään totuutta, mutta kun sinä luulet sellaisen jo omaavasi, kieltäydyt näkemästä sille vastakkaisia todisteita. Kerroin jo, että jääkaloille on syntynyt uusia ominaisuuksia ja rakenteita, kun ne todisteiden mukaan ovat kehittyneet noiden seutujen lämpimämpien vesien kaloista. Mistä puolestaan on todisteena mm. niiden rikkonaiset hemoglobiinigeenit. Käytit itse vastaavana esimerkkinä luolakaloja. Mutta sinä kieltäydyt hyväksymästä jääkaloja evoluution esimerkiksi vain ja ainoastaan siksi, että niille on kehittynyt uusia ominaisuuksia ja rakenteita evoluution keinoin. Jos ne olisivat pelkästään menettäneet hemoglobiinigeeninsä saisimme lukea sinulta jääkaloista lukuisissa viesteissä kuten luolakaloista tai siipensä menettäneistä koppakuoriaisista. Mutta koska niille on kehittynyt noita uusia ominaisuuksia mekanismeilla, jotka sinulle on jo kerrottu monesti, siirrytkin esittämään, että kyse olisikin vain perusteettomasta uskomuksesta menneisyydessä.

        "Joudutte tekemään näin siksi ettei evoluutiolla ole mekanismia kehittää uusia rakenteita ja siksi mikrobista ei ole voinut kehittyä mikään muu kuin mikrobi."

        Tietenkin evoluutiolla on mekanismit kehittää uutta. Nyt esim. laboratoriossa saatiin yksisoluisesta hiivasta kehittymään monisoluinen kanta. Ja mikrobista voi kehittyä jotain muuta kuin mikrobi, kun tapahtuu endosymbioosi. Todisteiden mukaan se on edellytys monimutkaisemmalle elämälle ja todisteiden mukaan sellaista on myös tapahtunut lukuisia kertoja eri kehityslinjoissa.

        Ottaen myös huomioon vanhan viestini, jossa kerroin sinulle kuinka Hawaiian Silversword Alliance on kehittynyt kantalajistaan n.50 eri lajiksi ja saanut uusilla geeneillään runsaastí uusia ominaisuuksia ja rakenteita, niin sinun esittämänäsi väite, ettei evoluutiolla olisi mekanismia kehittää uusia ominaisuuksia ja rakenteita on silkka valhe.


      • t-torppa kirjoitti:

        Evoluutioon uskovat opettavat että pelikaanit, granaattiomenat, ihmiset ja ponit ovat saaneet alkunsa yksisoluisista eliöistä (yksisoluisia eliöitä ovat esimerkiksi alkueläimet). Jokaisessa tapauksessa DNA- ”reseptin” on täytynyt läpikäydä valtava informaation lisääntyminen väitettyjen miljoonien vuosien aikana. Yksisoluisella eliöllä ei ole ohjeita silmien, korvien, veren, ihon, kavioiden, aivojen jne. valmistamiseen joita ponit tarvitsevat. Jotta ponit olisivat voineet saada alkunsa alkueläimistä täytyisi olla jokin mekanismi joka synnyttää uutta informaatiota.

        Evoluutioon uskovat tervehtivät luonnonvalintaa kuin se olisi luova jumalatar mutta tosiasia on että valinta sellaisenaan aina hankkiutuu eroon informaatiosta ei koskaan päinvastoin. Jotta olisi jokin tapa informaation lisäämiseksi evoluutioon tosiuskovien ainoa vaihtoehto ovat geneettiset kopiointivirheet tai vahingot eli sattumanvaraiset mutaatiot (joita valinta voi sitten ”suodattaa”). Ongelma on kuitenkin siinä että jos mutaatiot kykenisivät lisäämään tarvittavaa informaatiota meidän tulisi nähdä ympärillämme satoja esimerkkejä – ottaen huomioon, että jatkuvasti tapahtuu useita tuhansia mutaatioita. Mutta aina kun tutkimme mutaatioita, osoittautuu poikkeuksetta, että kyse on informaation menettämisestä tai huonontumisesta. Tämä on totta jopa niissä harvoissa tapauksissa joissa mutaation aiheuttama vika saa aikaan eloonjäämisedun – esimerkiksi kun tuulisilla saarilla elävät kovakuoriaiset menettävät siipensä. (katso lentokyvyttömät kovakuoriaiset - http://www.luominen.fi/genetiikka/lentokyvyttomat-kovakuoriaiset ).

        Eliöiden erilaistuessa geenistö hupenee lisääntyvässä määrin. Mitä enemmän eliöt sopeutuvat valinnan kautta ympäristöihinsä, eli mitä erikoistuneemmiksi ne tulevat, sitä pienempi osa niillä on jäljellä niiden luodulle lajille alun perin luodusta informaatiovarastosta. Siten luonnonvalinnan käytettävissä on vähemmän informaatiota tulevaisuudessa jotta se voisi uudelleen sopeutua jos olosuhteet muuttuisivat. Vähemmän mukautuvat, vähemmän sopeutumaan kykenevät populaatiot ovat selvästi kulkemassa kohti sukupuuttoa eivät kehittymässä.



        Muista että evoluutiouskomukset opettavat että olipa kerran eläviä olentoja mutta ei keuhkoja – keuhkot eivät olleet vielä kehittyneet joten ei ollut DNA-informaatiota joka olisi koodannut keuhkojen valmistamista. Jotenkin tämä ohjelma oli kirjoitettava. Oli synnyttävä uutta informaatiota jota ei aikaisemmin ollut olemassa - ei missään.

        Myöhemmin oli olemassa keuhkoja mutta missään maailmassa ei ollut höyheniä eikä siten geneettistä informaatiota höyhenille. Todellisen maailman tarkkailu on musertavalla tavalla osoittanut mutaation olevan täysin kykenemätön syöttämään järjestelmään tarvitun uuden informaation. Itse asiassa mutaatiot kiihdyttävät kokonaisuutena alaspäin menevää suuntausta lisäämällä geneettistä taakkaa vahingollisten mutaatioiden muodossa joita meihin kaikkiin on sukupolvien aikana kertynyt satamäärin.

        Toisin sanoen populaatiot voivat muuttua ja sopeutua koska niillä on paljon informaatiota (muuntelu kykyä) DNA- ”reseptissään”. Mutta elleivät mutaatiot kykene syöttämään perimään uutta informaatiota joka kerta kun muuntelua tai sopeutumista tapahtuu kokonaisinformaatio vähenee (kun valinta karsii sopeutumattomat osat populaatiosta osa informaatiosta menetetään tuossa populaatiossa).

        Siten kun informaatiota on tietty määrä niin mitä enemmän näemme sopeutumista sitä pienempi mahdollisuus sopeutumiseen on tulevaisuudessa. Evoluutiojuna suuntaa kulkunsa varmuudella alaspäin ja se on matkalla sukupuuton laituriin!

        "Muista että evoluutiouskomukset opettavat että olipa kerran eläviä olentoja mutta ei keuhkoja – keuhkot eivät olleet vielä kehittyneet joten ei ollut DNA-informaatiota joka olisi koodannut keuhkojen valmistamista. Jotenkin tämä ohjelma oli kirjoitettava. Oli synnyttävä uutta informaatiota jota ei aikaisemmin ollut olemassa - ei missään."

        Ja tutkimukset ovatkin paljastaneet, miten keuhkot kehittyivät alunperin: ne kehittyivät mutaatiosta, joka tuotti kaloille ruokatorveen laajentuman. Tuo laajentuma oli hyödyllinen, koska sen sisältämän ilmamäärän avulla kalat kykenivät säätelemään uimasyvyyttään ja vähähappisissa olosuhteissa kalat pystivät säilömään siinä ilmaa. Lopulta tuosta laajentumasta kehittyi muuntelun ja luonnonvalinnan avulla toisille kaloille uimarakko ja toisille keuhkot.

        http://www.news.cornell.edu/stories/2013/02/ct-scans-help-answer-question-fish-lung-evolution


      • t-torppa kirjoitti:

        Mitä uusia rakenteita uistinpakkiin syntyi? Jo olemassa oleva rakenne vain vääntyi - eli mitään uutta rakennetta ei syntynyt.

        Se että vaappu ui paremmin ja siksi saa etua muihin vaappuhin nähden ei tee vaappuun uusia monimutkaisia rakenteita jota evoluutio tarvitsisi jotta mikrobeista ihmiseksi kehitys olisi mahdollista.

        *Mitä uusia rakenteita uistinpakkiin syntyi? Jo olemassa oleva rakenne vain vääntyi - eli mitään uutta rakennetta ei syntynyt. *

        Kas, Torppa vastaa kun ei kysytä mutta ei vastaa kun kysytään. Selkärangattoman elkeitä.

        Mutta vastataan nyt kun on kysytty (minä en väistele kysymyksiä kuten sinä, Torppa): pakkiin olisi voinut muodostua yhtä hyvin viehe, jossa kaksi olemassaolevaa asiaa yhdistyy uudella tavalla. Esim. jigi jäänyt kiinni vaapun koukkuun, jolloin vaapusta tulee aivan uudenlainen viehe.

        Tässä vaiheessa kreationisti sanoo sitten että "se olisi edelleen vaappu".

        Aivan, ei sieltä pakista tule ulos pelkkiä jigejä tai vaappuja vaikka kuinka heiluttelisi, vaan ominaisuuksien uusia yhdistelmiä.


      • t-torppa kirjoitti:

        Evoluutiolla ei ole mekanismia ja tämä on sinulle jo monesti selitetty. Se että kilpa-auton säätöjä voidaan muunnella tietyissä variaatio rajoissa ei kerro mitään prosessista joka auton rakensi.

        Kaikki evolutionistien todisteet ovat vain tällaisia jo olemassa olevien rakenteiden muuntelua tietyissä variaatio rajoissa - teidän todisteet eivät kuitenkaan todista mitään miten rakenteet alunperin ovat saaneet alkunsa.

        Puhutte jo olemassa olevien rakenteiden ja ohjeiden säätelystä kun keskustelua käydään rakenteiden alkuperästä. Joudutte tekemään näin siksi ettei evoluutiolla ole mekanismia kehittää uusia rakenteita ja siksi mikrobista ei ole voinut kehittyä mikään muu kuin mikrobi.

        *Se että kilpa-auton säätöjä voidaan muunnella tietyissä variaatio rajoissa ei kerro mitään prosessista joka auton rakensi. *

        Sinulle on kerrottu että evoluution mekanismeja ovat mm. muuntelu / mutaatiot ja luonnonvalinta.

        Keropa Torppa miten ajattelet luonnonvalinnan vaikuttavan elottoman kilpa-auton menestymiseen lisääntyjänä?

        Vertauksesi on surkea, mutta sehän ei biologista evoluutiota ymmärtämätöntä uskovaista haittaa.


      • bg-ope kirjoitti:

        *Mitä uusia rakenteita uistinpakkiin syntyi? Jo olemassa oleva rakenne vain vääntyi - eli mitään uutta rakennetta ei syntynyt. *

        Kas, Torppa vastaa kun ei kysytä mutta ei vastaa kun kysytään. Selkärangattoman elkeitä.

        Mutta vastataan nyt kun on kysytty (minä en väistele kysymyksiä kuten sinä, Torppa): pakkiin olisi voinut muodostua yhtä hyvin viehe, jossa kaksi olemassaolevaa asiaa yhdistyy uudella tavalla. Esim. jigi jäänyt kiinni vaapun koukkuun, jolloin vaapusta tulee aivan uudenlainen viehe.

        Tässä vaiheessa kreationisti sanoo sitten että "se olisi edelleen vaappu".

        Aivan, ei sieltä pakista tule ulos pelkkiä jigejä tai vaappuja vaikka kuinka heiluttelisi, vaan ominaisuuksien uusia yhdistelmiä.

        Vaika kuinka yhdistelet mikrobin jo olemassa olevia ominaisuuksia et saa muuta aikaan kuin mikrobin. Jotta evoluutioteoria olisi totta tulisi pakkiin kehittyä täysin uusia rakenteita. Rakenteita joita ei ennen ollut olemassa missään.

        Jo olemassa olevien ominaisuuksien yhdisteleminen ei kerro mitään prosessista joka nämä ominaisuudet alkujaan synnytti jotenka et ole edes antanut esimerkkiä evoluutiosta.


      • t-torppa kirjoitti:

        Vaika kuinka yhdistelet mikrobin jo olemassa olevia ominaisuuksia et saa muuta aikaan kuin mikrobin. Jotta evoluutioteoria olisi totta tulisi pakkiin kehittyä täysin uusia rakenteita. Rakenteita joita ei ennen ollut olemassa missään.

        Jo olemassa olevien ominaisuuksien yhdisteleminen ei kerro mitään prosessista joka nämä ominaisuudet alkujaan synnytti jotenka et ole edes antanut esimerkkiä evoluutiosta.

        *Vaika kuinka yhdistelet mikrobin jo olemassa olevia ominaisuuksia et saa muuta aikaan kuin mikrobin. Jotta evoluutioteoria olisi totta tulisi pakkiin kehittyä täysin uusia rakenteita. Rakenteita joita ei ennen ollut olemassa missään. *

        Niin, siksi käytinkin esimerkkinä vaappuja ja jigejä. Kun ne yhdistyvät, syntyy jotain uutta, täysin uusia rakenteita. Meneekö yli hilseen?

        Kerrotaanpa siis pari esimerkkiä reaalimaailmasta: kun alkeellisen solukalvopalluran sisälle ajautuu solusyönnin kautta fotosynteesiin kykenevän bakteeri, syntyy leväsolu. Kun alkeellisen solukalvopalluran sisälle ajautuu solusyönnin kautta soluhengitykseen kykenevä bakteeri, syntyy toisenvaraisen eläinsolun aihe. Kyseinen ilmiö on nimeltään endosymbioosi, ja se on todennettavissa sillä että sekä kasvisolujen viherhiukkaset että eläinsolun mitokondriot muistuttavat edelleen ominaisuuksiltaan erittäin paljon bakteereja.
        http://fi.m.wikipedia.org/wiki/Endosymbioositeoria

        *Jo olemassa olevien ominaisuuksien yhdisteleminen ei kerro mitään prosessista joka nämä ominaisuudet alkujaan synnytti jotenka et ole edes antanut esimerkkiä evoluutiosta.*

        Luit toivon mukaan juuri teorian siitä, miten osa soluorganelleista on päätynyt soluun. Olet tervetullut esittämään vaihtoehtoisia prosesseja, näytön kera.

        Ja mitä tulee evoluutioon ja sen esimerkkeihin, se että lapsi ei ole vanhempiensa klooni riittää. Koska se on muuntelua, ja muuntelu evoluutiota.

        Saimaannorppa, itämerennorppa ja laatokannorppa lajiutuvat sukupolvi sukupolvelta, etkö sinä valheinesi mahda sille(kään) kehitykselle yhtään mitään. Kaikki maailman ala-lajit ovat matkalla omiksi itsenäisiksi lajeikseen, sinulta lupia kyselemättä.


    • Totuus on armoton

      Jos kuvitellaan että hevonen on kehittynyt eohippuksesta, eli tuosta vähän rottaa suuremmasta jyrsijästä, niin kaikki välimuodot täytyy kuvitella, eikä mielikuvitusolentoja voida tutkia millään tieteen menetelmällä, joten evoluutio on todellisuutta siihen uskovan päässä, ei muualla, sillä noita kuviteltuja välimuotoja ei ole nähty.

      • *Jos kuvitellaan että hevonen on kehittynyt eohippuksesta, eli tuosta vähän rottaa suuremmasta jyrsijästä, niin kaikki välimuodot täytyy kuvitella, eikä mielikuvitusolentoja voida tutkia millään tieteen menetelmällä, joten evoluutio on todellisuutta siihen uskovan päässä, ei muualla, sillä noita kuviteltuja välimuotoja ei ole nähty.*

        Jaa, luonnontieteellisissä museoissa säilytettävät sadat tuhannet fossiilinäytteet ovat siis kaikki kuvitteellisia. Kiitos tuosta henkilökohtaiseen mielipiteeseesi perustuvasta varmasta tiedosta.

        Voitko kertoa keneltä lähetän terveisiä ja kerron tuon kirjoittamasi totuuden kun menen niitä seuraavan kerran katsomaan? Huikkaan siinä samalla sinne labran puolelle että "kuvitteellisia asioita te siellä leikkelette".


    Ketjusta on poistettu 1 sääntöjenvastaista viestiä.

    Luetuimmat keskustelut

    1. KUPSinpelaaja vangittu törkeästä rikoksesta

      Tänään tuli uutinen että Kupsin sopimuspelaajs vangittu törkeästä rikoksesta epäiltynä. Kuka pelaaja kysressä ja mikä ri
      Kuopio
      19
      1651
    2. Taasko se show alkaa

      Koo osottaa taas mieltään
      Ikävä
      29
      1319
    3. Minun oma kaivattuni

      Ei ole mikään ilkeä kiusaajatyyppi, vaan sivistynyt ja fiksu sekä ystävällinen ihminen, ja arvostan häntä suuresti. Raka
      Ikävä
      64
      1224
    4. Miksi ihmeessä nainen seurustelit kanssani joskus

      Olin ruma silloin ja nykyisin vielä rumempi En voi kuin miettiä että miksi Olitko vain rikki edellisestä suhteesta ja ha
      Ikävä
      11
      1122
    5. Persut nimittivät kummeli-hahmon valtiosihteeriksi!

      Persujen riveistä löytyi taas uusi törkyturpa valtiosihteeriksi! Jutun perusteella järjenjuoksu on kuin sketsihahmolla.
      Perussuomalaiset
      28
      1066
    6. Tervehdys!

      Sä voit poistaa nää kaikki, mut mä kysyn silti A:lta sen kokemuksia sun käytöksestä eron jälkeen. Btw, miks haluut sabot
      Turku
      65
      1046
    7. Onko ministeri Juuso epäkelpo ministerin tehtäviensä hoitamiseen?

      Eikö hänellä ole kompetenttia hoitaa sosiaali- ja terveysministetin toimialalle kuuluvia ministerin tehtäviä?
      Perussuomalaiset
      11
      1036
    8. Elia tulee vielä

      Johannes Kastaja oli Elia, mutta Jeesus sanoi, että Elia tulee vielä. Malakian kirjan profetia Eliasta toteutuu kokonaan
      Helluntailaisuus
      30
      1009
    9. Sakarjan kirjan 6. luku

      Jolla korva on, se kuulkoon. Sain profetian 22.4.2023. Sen sisältö oli seuraava: Suomeen tulee nälänhätä niin, että se
      Profetiat
      6
      1001
    10. Kaupungin valtuuston yleisötilaisuus

      YouTubessa katsojia 76 Buahahaha buahahaha buahahaha buahahaha buahahaha buahahaha
      Varkaus
      2
      996
    Aihe